BCSE 15 Days For Test Day Flashcards

1
Q

Which one of the following choices best describes the use of Digital Imaging and Communication in Medicine (DICOM) in digital radiology?
A universal digital image format that allows sharing of images between clinics
A technique that transforms digital images into hard copies for storage purposes
A method of obtaining digital radiographs with less radiation exposure
A means of converting traditional x-ray film into digital images
A mechanism that prevents artifacts from occurring on digital images

A

Answer: A universal digital image format that allows sharing of images between clinics.

Digital Imaging and Communication in Medicine (DICOM) is a universal digital image format that allows sharing of electronic images with hospitals, specialists and other locations (teleradiology).

DICOM is the standard image format for medical image generation and storage.

DICOM allows storage of images from a variety of different modalities (e.g., digital and computed radiography, computed tomography (CT), MRI, ultrasonography, digital fluoroscopy).

The DICOM image file contains patient identification, date and time of the examination, and display formats.

Link: Radiography on Animals. Filmless Radiography for Diagnostic Imaging in Animals.
https://www.merckvetmanual.com/clinical-pathology-and-procedures/diagnostic-imaging/radiography-of-animals?redirectid=4195?ruleredirectid=30#v3298717

Filmless Radiography in Animals

Overview
Filmless radiography, also known as digital radiography, has become the standard in veterinary diagnostic imaging due to its numerous advantages over traditional film-based methods. Digital radiography eliminates the need for film, screens, and processing chemicals, and offers several significant benefits for veterinary practices.

Types of Digital Radiography
1. Computed Radiography (CR)
• Mechanism: Uses a semiconductor plate in a cassette, exposed in the same manner as traditional film, which is then read electronically by a special reader.
• Advantages: Portable cassettes, cost-effective compared to DR systems, high resolution, and suitable for multiple imaging locations.
• Disadvantages: Requires periodic replacement of plates due to wear, intermediate reading step can be cumbersome.
2. Direct Digital Radiography (DR)
• Mechanism: Utilizes a cesium iodide scintillator array to absorb x-rays, which are then detected by an array of photodiode/transistor elements to create a digital image.
• Advantages: Faster image acquisition (available within 30 seconds), no need for intermediate reading steps, reduced wear and tear, better suited for high-volume practices.
• Disadvantages: Higher initial cost, sensitivity to electronic interference and physical shock.

How well did you know this?
1
Not at all
2
3
4
5
Perfectly
2
Q

Abdominal ultrasound of a dachshund reveals bilaterally enlarged adrenal glands (an example of one of the images is shown below). What clinical signs would you expect to see in a dog with these findings?
Lethargy, tragic facial expression, tail alopecia
Pruritis, alopecia of lower back, cold intolerance
Polydipsia, seizures, watery diarrhea
Polyuria, pendulous abdomen, truncal alopecia
Gastroenteritis, bradycardia, exercise intolerance

A

Answer: Polyuria, pendulous abdomen, truncal alopecia.

This is likely to be pituitary hyperadrenocorticism (HAC). Signs might include polyuria, pendulous abdomen, truncal alopecia.

This dog might also have polydipsia, polyphagia, muscle weakness, and recurrent urinary tract infections.

Roughly 80%-85% of canine HAC cases are pituitary-expect bilateral enlargement greater than 8mm on ultrasound.

Roughly 15%-20% of canine HAC cases are adrenal-expect unilateral adrenal enlargement greater than 8mm, with possible atrophy in the contralateral gland.

You can also see unilateral adrenomegaly with an incidental, nonfunctional adrenal mass or due to pheochromocytoma. This disease is relatively rare and difficult to diagnose.

Signs are non-specific, but intermittent weakness and collapse are commonly reported in diagnosed case-patients

Tail alopecia, tragic facial expression, and lethargy are signs of hypothyroidism.

Refs: Cote, Clinical Veterinary Advisor-Dogs and Cats, 3rd ed. pp. 525-7. Image courtesy, Dr Kalumet.

Link: Cushing Syndrome
https://www.merckvetmanual.com/endocrine-system/the-adrenal-glands/cushing-syndrome-hyperadrenocorticism

Link: Pheochromocytomas
https://www.merckvetmanual.com/endocrine-system/neuroendocrine-tumors/pheochromocytomas

Link: Hypothyroidism in Animals
https://www.merckvetmanual.com/endocrine-system/the-thyroid-gland/hypothyroidism-in-animals

How well did you know this?
1
Not at all
2
3
4
5
Perfectly
3
Q

A five-year-old female spayed domestic shorthair cat is presented with a month-long history of anorexia and lethargy. On physical examination she is 5-7% dehydrated. Which one of the following choices is apparent form these thoracic radiographs?

Hypovolemia
Normal radiographs
Hypertrophic cardiomyopathy
Allergic pneumonitis
Heartworm diseases

A

Answer: Hypovolemia.

This cat has hypovolemia (secondary to hepatic lipidosis).
The heart is small and lifted off of the sternum. The pulmonary vasculature, aorta, and caudal vena cava are reduced in size.

The pulmonary structures appear within normal limits. Within the viewable abdomen the liver appears enlarged, extending past the costochondral arches.

When circulating blood volume is reduced through blood loss, dehydration, or shock, the cardiac and vascular structures in the thorax can appear on radiographs.

In this case, the heart is small, giving the appearance of lifting from the sternum. The heart often appears more angular and less rounded than usual. The reduced pulmonary circulation also makes the lungs appear more lucent.

In this case, hepatic lipidosis (causing the apparent hepatomegaly) was the primary disease with secondary dehydration.

Link: Feline Hepatic Lipidosis
https://www.merckvetmanual.com/digestive-system/hepatic-disease-in-small-animals/feline-hepatic-lipidosis

Link: Normal Feline Thoracic Radiographs
https://vetmed.illinois.edu/imaging_anatomy/feline/thorax/ex03/thor03-f%20.html

How well did you know this?
1
Not at all
2
3
4
5
Perfectly
4
Q

A draft horse is presented whose left forefoot radiograph looks like the image below. What is the diagnosis?
Sidebone
Low ringbone
Navicular syndrome
Osselets
High ringbone

A

Answer: Sidebone.

This is an example of sidebone, an ossification of the cartilages of the third phalanx seen most often in heavy working horses.

Navicular syndrome (also called caudal heel pain) involves the navicular bone (distal sesamoid) and looks like this on radiographs (note the lollipops).

Periostitis & osteoarthritis of the proximal or distal interphalangeal joints leading to exostoses is called ringbone (high or low). Usually brought on by faulty conformation, improper shoeing, repeated concussion on hard surfaces, trauma, and infection.

Osselets is ringbone (arthritis, periosteitis) around the metacarpal/metatarsal phalangeal joint (fetlock).

Image courtesy of Nora Grenager, VMD, DACVIM.

Link: Sidebone in Horses
https://www.merckvetmanual.com/musculoskeletal-system/lameness-in-horses/sidebone-in-horses

Link: Navicular Disease in Horses
https://www.merckvetmanual.com/musculoskeletal-system/lameness-in-horses/navicular-disease-in-horses

Link: Osteoarthritis of the Proximal Interphalangeal Joint in Horses
https://www.merckvetmanual.com/musculoskeletal-system/lameness-in-horses/osteoarthritis-of-the-proximal-interphalangeal-joint-in-horses

How well did you know this?
1
Not at all
2
3
4
5
Perfectly
5
Q

A twelve-year-old female spayed rottweiler is presented for an acute onset of difficulty breathing. For the past month the dog has been lethargic with decreased appetite and weight loss.
A lateral thoracic radiograph is shown below.
What is the most likely diagnosis?

Left-sided heart failure
Aspiration pneumonia
Pneumothorax
Lymphoma
Diaphragmatic hernia

A

Answer: Lymphoma

This dog has pleural effusion caused by intrathoracic lymphoma.
Note the soft tissue density in the ventral chest with rounded lung lobe margins and widened interlobar fissures consistent with pleural effusion. The cardiac silhouette is obscured.
Confirm the presence of effusion with thoracic focused assessment with sonography for triage, trauma, and tracking (TFAST).
Perform thoracocentesis for both diagnostic and therapeutic purposes. Submit fluid for analysis and cytology to potentially identify neoplastic cells. If fluid cytology does not reveal an etiology, consider further imaging of the chest such as thoracic ultrasound or CT scan.

CGPT: Comprehensive Summary on Lymphoma in Dogs

How well did you know this?
1
Not at all
2
3
4
5
Perfectly
6
Q

A guinea pig is presented with inappetence, oculonasal discharge, and dyspnea. The clients also have a pet rat, and a pair of rabbits, and these animals all have frequent contact with each other.
Which of the following agents is the most likely cause?

Staphylococcus aureus
Dromaius novaehollandiae
Hymenolepis diminuta
Clostridium difficile
Bordetella bronchiseptica

A

Answer: Bordetella bronchiseptica

Rabbits can be subclinical carriers of Bordetella bronchiseptica, which is highly_pathogenic in guinea pigs. Because of this cross-species pathogenicity, rabbits and guinea pigs should not be kept together as pets.
Hymenolepis diminuta is a dwarf tapeworm in gerbils. It is possibly zoonotic to humans.

CGPT: Comprehensive Summary on Guinea Pigs

CGPT: Comprehensive Summary on Gerbils

How well did you know this?
1
Not at all
2
3
4
5
Perfectly
7
Q

A two-week-old calf is presented for colic and hemorrhagic diarrhea. It is one of the fastest-growing calves in the pasture and was normal yesterday.
The calf has abdominal distention, cyanotic mucous membranes, an upper eyelid skin tent of five seconds, and eyeball recession of six mm. During the physical exam the calf collapses and shows signs of opisthotonus.
What is the most likely diagnosis?

Coccidiosis
Cryptosporidiosis
Enterotoxemia
Coronavirus
Salmonellosis

A

Answer: Enterotoxemia

Peracute colic, hemorrhagic diarrhea, and neurologic signs in a two-week-old fast growing calf is a classic presentation for enterotoxemia (Clostridium perfringens type C). The findings of cyanosis along with skin tent and eyeball recession indicate moderate to severe dehydration and cardiovascular compromise.
Treatment consists of fluid therapy, Clostridium perfringens type C antitoxin, and antimicrobials. Oral penicillin can be used in food animals. Oral metronidazole is recommended in horses but is prohibited for use in food animals in the U.S.
Prognosis is poor even with treatment so best to prevent through vaccination of dams and subsequent passive transfer of immunity to calves.
Colibacillosis is a possible cause of peracute diarrhea but is typically seen in very young calves less than five days of age.
In general, remember the AGE, SEVERITY, and # AFFECTED for calf diarrheas.
The EARLY diarrheas are due to:
Escherichia coll - less than 5d old, multiple cases Rotavirus - 5d-2 wks old, self limiting, multiple calves Coronavirus - 4-30d, multiple calves
Enterotoxemia - (Clostridium perfringens type C) 2 wks old, healthiest, fastest growing calf, hemorrhagic diarrhea, sudden death
Cryptosporidia -1-4 wks, diarrhea, tenesmus, emaciation
LATER diarrheas are:
Neonatal salmonella - 1-2 mos, fever and diarrhea, septicemia, sudden death
Coccidiosis - older than 21 days and in ALL ages, bloody diarrhea, tenesmus

CGPT: Enterotoxemias in Animals

CGPT: Diarrhea in Neonatal Ruminants

CGPT: Cryptosporidiosis in Animals

CGPT: Salmonellosis in Animals

CGPT: Coccidiosis of Cattle

How well did you know this?
1
Not at all
2
3
4
5
Perfectly
8
Q

A cow showing chronic cerebral neurologic signs is presented for euthanasia.
You submit the brain for histopathology. Microscopically the brain tissue appears vacuolated (see image below).
Which one of the following choices is the most likely diagnosis?

Malignant catarrhal fever
Rabies
Bovine spongiform encephalopathy
Actinomycosis
Haemophilus somnus thromboembolic meninogencephalitis

A

Answer: Bovine spongiform encephalopathy

These are the typical histopathology findings highly suggestive of bovine spongiform encephalopathy. (BSE). The misfolded proteins (prions) cause vacuolization (“spongiosis”) of the brain.
BSE causes variant Creutzfeldt-Jakob disease (vCJD) in humans
(ZOONOTIC) and a progressive, fatal, infectious neurologic disease of the central nervous system of cattle.
Contact the proper regulatory personnel (REPORTABLE) while the lab performs a confirmatory diagnostic test (e.g., immunohistochemistry, Western immunoblot).
A characteristic histopathologic appearance of rabies is the presence of intracytoplasmic inclusions known as Negri bodies.

CGPT: Bovine Spongiform Encephalopathy (BSE)

How well did you know this?
1
Not at all
2
3
4
5
Perfectly
9
Q

The best place to take a horse’s pulse is to place fingers over
which one of the following structures?

Vena Cava
Jugular vein
Facial artery
Carpus
Heart

A

Answer: Facial Artery.

The most convenient location for taking the pulse of a horse is to place two or three fingers over the facial artery, in the notch of the mandible (lower jaw) just in front of the masseter (cheek) muscle.
Click here to see a to see a video of a vet taking a horse’s pulse.
Click here to see a Table of normal heart-rates of different animals.
Table of normal respiration-rates of different animals.

How well did you know this?
1
Not at all
2
3
4
5
Perfectly
10
Q

Which one of the following choices can be added to the diet of sheep and goats to reduce the incidence of frothy bloat?

lonophores
Vitamin E
Antiparasitics
Sulfa antibiotics
Minerals

A

Answer: Ionophores

The inclusion of ionophores in the diet decreases the formation of free ruminal gas, which can then get trapped in a stable foam leading to frothy bloat.
Frothy bloat occurs when chloroplast proteins liberated by digestion coat gas bubbles produced normally by rumen microbes.
The bubbles become stabilized by the proteinaceous froth. The animal cannot eructate the froth, which leads to rumen tympany.
The froth also cannot pass through a needle or a stomach tube.
Frothy bloat usually occurs after the ingestion of a lot of legumes (e.g., clover, alfalfa), and/or ingestion of lush pasture. Slow introduction to these feedstuffs, providing supplementatal feed, and feeding poloxalene can also help prevent.

CGPT: Bloat in Ruminants

How well did you know this?
1
Not at all
2
3
4
5
Perfectly
11
Q

Pediculosis is correctly described by which one of the following choices?

Invasion of fly larvae
Colonization with protozoa
Infection of a dog’s paw
Infestation with lice

A

Answer: Infestation with lice

CGPT: Lice in Animals

How well did you know this?
1
Not at all
2
3
4
5
Perfectly
12
Q

Which one of the following choices is the best way to prevent o
grain overload?

Ensure adequate water supply
Transfaunate the rumen
Give penicillin when concentrates are increased
Introduce dietary changes slowly
Restrict movement and access to legume hay

A

Answer: Introduce dietary changes slowly.
Grain overload results when ruminants ingest too much feed with readily digestible carbohydrates at one time.
Excessive carbohydrate causes a shift to gram-positive organisms in the rumen, especially Streptococcus bovis, which proliferate excessively.
These organisms produce large amounts of acid which damage the rumen and are absorbed, causing acidosis, dehydration, ileus, and eventually cardiovascular collapse and death.
Transfaunation of the rumen fluid is helpful after grain overload is resolved to help restore the normal flora of the rumen.

CGPT: Grain Overload in Ruminants

How well did you know this?
1
Not at all
2
3
4
5
Perfectly
13
Q

According to current American Association of Feline Practitioners (AAFP) recommendations, which three vaccines should pet cats routinely receive in addition to rabies and feline leukemia vaccinations?

Intranasal feline herpesvirus, feline parvovirus,
pneumonitis
Panleukopenia, feline herpesvirus-1, calicivirus
Bordatella bronchiseptica, feline immunodeficiency virus, dermatophyte
Feline infectious peritonitis, dermatophyte, calicivirus
Feline viral rhinotracheitis, Mycoplasma felis,
Chlamydophilia felis

A

Answer: Panleukopenia, feline herpesvirus-1, calicivirus

The American Association of Feline Practitioners (AAFP) vaccination guidelines recommend three “core” vaccines for ALL cats, +/- other vaccines depending on the circumstances (i.e.: household pet, shelter).
The three core feline vaccines are:
1. Feline herpesvirus-1 ( FHV-1)
2. Feline panleukopenia (feline parvovirus, FPV)
3. Feline calicivirus (FCV)
The AAFP states that rabies is “essential in regions where required by statute/law,” and is required in most states in the US and Canada. So, cats should receive rabies vaccination in addition to the core three.
Other vaccines to consider, depending on circumstances:
- Feline leukemia virus (FeLV) vaccination is recommended for group-housed cats and is considered a core vaccine for pet cats <1
yo
- For shelters consider intranasal FHV-1 and FCV on admission for high-risk populations and kittens >4 wo. Provides more rapid protection.
Here are the 2020 AAFP feline vaccination guidelines.

https://journals.sagepub.com/doi/pdf/10.1177/1098612X20941784

CGPT: Feline Panleukopenia

CGPT: Feline Respiratory Disease Complex

CGPT: Feline Leukemia Virus (FeLV)

How well did you know this?
1
Not at all
2
3
4
5
Perfectly
14
Q

Which one of the following diseases/syndromes is NOT reportable?
Avian psittacosis (Chlamydia psittaci)
Johne’s disease
Pthiris pubis infestation
Any vesicular disease of livestock

A

Answer: Pthiris pubis infestation.

Although the presence of Pthiris pubis (Human pubic lice) on a pet may be alarming to the owner, human pubic lice are NOT reportable. Psittacosis IS reportable because it is infectious and a serious cause of zoonotic respiratory disease in birds and humans.

Johne’s disease IS reportable in cattle. Brucellosis IS REPORTABLE. Basically, ANY vesicular disease of livestock IS REPORTABLE.

There are a number of things you should do in the face of a vesicular disease outbreak,
but the most important is to report the incident to the state vet/federal authorities.

The most important rule out is Foot and Mouth disease, which is difficult to distinguish from Swine vesicular disease, Vesicular exanthema of swine, and Vesicular stomatitis.

Because FMD is NOT present in the US, suspicion of it in a domestic swine herd is cause for state/federal government concern because of the potential agro-economic consequences. DDX via blood tests, primarily PC and biopsy.

Refs: Merck Veterinary Manual online edition.

Link: Avian Chlamydiosis
https://www.merckvetmanual.com/poultry/avian-chlamydiosis/avian-chlamydiosis

Link: Paratuberculosis in Ruminants
https://www.merckvetmanual.com/digestive-system/intestinal-diseases-in-ruminants/paratuberculosis-in-ruminants?redirectid=3156?ruleredirectid=30

Link: Overview of Brucellosis in Large Animals
https://www.merckvetmanual.com/reproductive-system/brucellosis-in-large-animals/overview-of-brucellosis-in-large-animals

Link: Foot-and-Mouth disease in Animals
https://www.merckvetmanual.com/generalized-conditions/foot-and-mouth-disease/foot-and-mouth-disease-in-animals?autoredirectid=14225

Link: Swine Vesicular Disease
https://www.merckvetmanual.com/generalized-conditions/swine-vesicular-disease/swine-vesicular-disease?autoredirectid=16670

Link: Vesicular Exanthema of Swine
https://www.merckvetmanual.com/generalized-conditions/vesicular-exanthema-of-swine/vesicular-exanthema-of-swine?autoredirectid=14240

Link: Vesicular Stomatitis in Horses
https://www.merckvetmanual.com/horse-owners/disorders-affecting-multiple-body-systems-of-horses/vesicular-stomatitis-in-horses

How well did you know this?
1
Not at all
2
3
4
5
Perfectly
15
Q

Which of the following is the correct description of pre-emptive analgesia?
Use of an antagonist to reverse dangerous side effects of an analgesic medication
The combination of different types of analgesic medications and procedures to treat pain
Treatment with pain medication during surgery and prior to recovery of a patient
Administration of pan medication prior to elective surgery and stimulation of the pain response
Use of higher doses of pain medication given previously to a patient to achieve adequate analgesia

A

Answer: Administration of pain medication prior to elective surgery and stimulation of the pain response.

Administration of pain medication prior to elective surgery / stimulation of the pain response is preemptive analgesia. Giving analgesics before the pain starts is the best way to minimize the pain response in a healthy patient who is not painful prior to surgery.

Fewer complications and shorter hospital stays are seen when patients are comfortable following painful procedures.

Analgesic therapy should be continued perioperatively and postoperatively for a few days to weeks depending on the procedure. Dosing should be decreased gradually and then stopped.

Tolerance is seen when higher doses of pain medications are required. This occurs with opioids. Using combinations of different types of analgesic medications and procedures is multi- modal analgesia/anesthesia.

Better analgesia and sedation are usually seen with combination therapy; lower doses can often be utilized as synergistic effects are common.

Click here to see a good summary on pain management, courtesy of the American College of Veterinary Surgeons (ACVS).
https://www.acvs.org/small-animal/pain-management

Refs: Bassert, Beal and Samples, McCurnin’s Clinical Textbook for Veterinary Technicians, 9th ed., p. 988, Gaynor & Muir Handbook of Vet Pain Mgt 2nd ed. pp.57, 349, 482, Tranquilli, Grimm, & Lamont Pain Mgt for the SA Practitioner 2nd ed. p. 10 and Greene’s Vet Anes & Pain Management Secrets, pp. 331-3.

Link: Small Animal Health Topics
https://www.acvs.org/small-animal

How well did you know this?
1
Not at all
2
3
4
5
Perfectly
16
Q

For what condition is dogs is the parotid duct sometimes translocated to the lateral canthus of the eye?
Imperforate lacrimal puncta
Refractive glaucoma
Diabetes-associated cataracts
Cherry eye
Keratoconjunctivitis sicca

A

Answer: Keratoconjunctivitis sicca.

Parotid duct translocation is indicated for chronic, nonresponsive cases of keratoconjunctivitis sicca (KCS). The parotid gland then lubricates the eye with saliva.

KCS is due to aqueous tear deficiency in dogs (and less so, cats and horses). Causes in dogs: Autoimmune dacryoadenitis (most common), distemper, idiopathic (removal of nictitans), drug-induced (NSAIDs and sulfonamides), neurogenic, traumatic, or congenital.

Clinical signs: Persistent, painful mucopurulent conjunctivitis with corneal ulceration and scarring. Corneas look dull.

Medical Tx: Topical artificial tears and antibiotic-corticosteroid combinations (if no corneal ulceration); increase tear production with topical cyclosporine A, tacrolimus, or pimecrolimus. Can give enteral pilocarpine for neurogenic KCS and topical acetylcysteine to lyse excess mucus and allow other topical medications to spread.

Lifelong Tx typically necessary.

Click here to see images and read more on KCS from the American College of Veterinary Ophthalmology.
https://www.acvo.org/common-conditions-1/2018/2/2/dry-eye-keratoconjunctivitis-sicca

Refs: The American College of Veterinary Ophthalmology website, Côté, Clin. Vet. Advisor: Dogs and Cats, 3rd ed. pp. 577-8,.

Link: Nasolacrimal and Lacrimal Apparatus
https://www.merckvetmanual.com/eye-diseases-and-disorders/ophthalmology/nasolacrimal-and-lacrimal-apparatus?redirectid=27591

How well did you know this?
1
Not at all
2
3
4
5
Perfectly
17
Q

Which one of the following is true regarding bovine leukosis virus?
Most infected animals have no outward clinical signs
The mosto common consequence of infection is lymphosarcoma
Feces of infected cattle are a major source of transmission
Passage in milk has been linked to human adult T cell leukemia / lymphoma
Beef cattle are more frequently infected that dairy cattle

A

Answer: Most infected animals have no outward clinical signs.

The most common outcome of infection with Bovine leukosis virus (BLV) is persistent infection with no outward clinical signs or lymphocytosis.

About a 1/3 of infected cattle develop a persistent benign lymphocytosis and <5% develop lymphosarcoma.

BLV infection is more common in dairy cattle than in beef cattle.

BLV is not a zoonosis. There is no treatment.

Risk is minimized by preventing the transfer of blood (and therefore infected lymphocytes) between cattle. Vertical transmission can also occur.

Refs: The Merck Veterinary Manual online edition.

Link: Overview of Bovine Leukosis
https://www.merckvetmanual.com/generalized-conditions/bovine-leukosis/overview-of-bovine-leukosis

Bovine Leukosis: Comprehensive Information for BCSE Test

Definition
• Bovine Leukosis (Bovine Lymphosarcoma): A cancerous disease caused by Bovine Leukemia Virus (BLV), an oncogenic retrovirus.

Etiology and Transmission
• Causative Agent: Bovine Leukemia Virus (BLV), a C-type oncogenic retrovirus.
• Transmission: Primarily horizontal through blood transfer (e.g., contaminated needles, dehorning tools), and rarely vertical (transplacental or via colostrum).

Epidemiology
• Prevalence: High in US dairy herds (~46.5%) and lower in beef herds (~10.3%).
Pathogenesis
• Infection Outcomes:
• Asymptomatic: Most animals show no signs.
• Persistent Lymphocytosis: ~29% develop this benign condition.
• Lymphosarcoma: <5% develop cancerous tumors.
Clinical Findings
• Juvenile Lymphosarcoma: Weight loss, fever, dyspnea, bloat, and posterior paresis.
• Thymic Lymphosarcoma: Cervical swelling, dyspnea, bloat, jugular distention, tachycardia.
• Cutaneous Lymphosarcoma: Cutaneous plaques, enlarged lymph nodes.
Diagnosis
• Serology: ELISA and PCR for BLV detection.
• Cytology/Histology: Biopsy for diagnosing lymphosarcoma.
Treatment and Control
• No Treatment: No effective treatment; symptomatic care with corticosteroids.
• Control Measures: Testing and culling infected animals, disinfection of equipment, using single-use needles.

Prevention
• Blood Precautions: Avoiding exposure to blood from infected animals.
• Colostrum Management: Using colostrum from seronegative cows.
• Farm Practices: Cleaning equipment and facilities, controlling flies.
Zoonotic Risk
• Human Risk: Potential, but not well-established, zoonotic risk through ingestion of unpasteurized dairy products.

How well did you know this?
1
Not at all
2
3
4
5
Perfectly
18
Q

Which of the following opioids is more potent than morphine, given most often via continuous intravenous infusion, or can be administered via transdermal patch?
Fentanyl
Naloxone
Oxymorphone
Buprenorphine
Tramadol

A

Answer: Fentanyl.

Fentanyl is very lipophilic, so it is absorbed and eliminated quickly. It is therefore not useful for pain control via single doses given intramuscularly (IM) or intravenously (IV).

However, it produces good analgesia when given continuously via intravenous infusion and, in many patients, via a transdermal patch. Serum levels can be variable in some patients with patches.

Absorption is affected by temperature; the use of heating blankets can increase levels, hypothermia will decrease levels.

Fentanyl is less likely to cause nausea and vomiting compared to morphine and other mu-agonist opioids. It does cause respiratory depression and dysphoria in some patients.

Other analgesics that can be administered via transdermal patches are lidocaine and buprenorphine.

Use of patches in patients at home must be evaluated carefully - toxicity and death has been seen in children and small animals that have ingested fentanyl patches.
See very nice articles on pain management and opioids in small animals: Murrell J. Clinical use of opioids in dogs and cats: Part 1. Companion Animal 2011; 16(4): 35-8, and Part 2, 16(5): 44-9.

The American Animal Hospital Association (AHA) provides this chart on Opioids and their actions
https://www.aaha.org/aaha-guidelines/2020-aaha-anesthesia-and-monitoring-guidelines-for-dogs-and-cats/drug-combinations/opiods/

Refs: Bassert, Beal and Samples, McCurnin’s Clinical Textbook for Veterinary Technicians, 9th ed., p. 993, Gaynor & Muir Handbook of Vet Pain Mgt 2nd ed. pp. 118-9, 244-5, 169, 348, 422, Greene, Vet Anes & Pain Mgt Secrets pp. 78-9, 336-7.

Link: Analgesic Pharmacology
https://www.merckvetmanual.com/management-and-nutrition/pain-assessment-and-management/analgesic-pharmacology

Comprehensive Summary on Analgesics Used in Animals for BCSE Test Preparation
Opioids
• Mechanism: Bind to opioid receptors (mu, kappa, delta) altering pain perception.
• Examples:
• Morphine, Hydromorphone, Fentanyl: Strong mu agonists.
• Buprenorphine: Partial mu agonist.
• Butorphanol: Agonist-antagonist.
• Effects: Analgesia, sedation, euphoria, dysphoria, respiratory depression.
Nonsteroidal Anti-inflammatory Drugs (NSAIDs) and Corticosteroids
• NSAIDs:
• Mechanism: Inhibit COX enzymes, reducing prostaglandin synthesis.
• Examples: Meloxicam, Robenacoxib (cats), Carprofen, Deracoxib (dogs).
• Effects: Analgesia, anti-inflammatory, antipyretic.
• Corticosteroids:
• Mechanism: Inhibit phospholipase A2, reducing inflammation.
• Examples: Prednisone, Dexamethasone.
• Effects: Anti-inflammatory, immunosuppressive, analgesic.
Alpha2-Agonists
• Mechanism: Bind to alpha2-adrenergic receptors, providing analgesia and sedation.
• Examples: Xylazine, Dexmedetomidine.
• Effects: Sedation, analgesia, muscle relaxation.
• Risks: Bradycardia, hypotension, arrhythmias.
Ketamine
• Mechanism: NMDA receptor antagonist, preventing central sensitization.
• Effects: Somatic analgesia, poor visceral analgesia, prevents wind-up.
• Uses: As part of multimodal analgesia.
Other Analgesic Agents
• Tramadol:
• Mechanism: Weak mu agonist, inhibits serotonin and norepinephrine reuptake.
• Effects: Analgesia, anti-anxiety.
• Risks: Seizure threshold reduction.
• Gabapentin:
• Mechanism: Binds to alpha2-delta subunit of voltage-gated calcium channels.
• Uses: Neuropathic pain, chronic pain.
• Amantadine:
• Mechanism: NMDA receptor antagonist.
• Uses: Chronic pain, opioid tolerance.
• Acetaminophen:
• Mechanism: Unclear, possibly COX-3 inhibition.
• Uses: Breakthrough pain in dogs (not cats due to toxicity).
• Maropitant:
• Mechanism: NK-1 receptor antagonist.
• Uses: Anti-emetic, visceral analgesia.
• Monoclonal Antibodies:
• Examples: Frunevetmab (for osteoarthritis in cats).
• Mechanism: Target NGF, reducing pain.

How well did you know this?
1
Not at all
2
3
4
5
Perfectly
19
Q

Which part of a tooth is the occlusal surface?
Towards the roof of the mouth
Toward the cheek
Toward the tongue
Chewing surface
Above the gum line

A

Answer: Chewing surface.

The occlusal surface of the tooth is the chewing surface. The crown is the visible tooth above the gum line. Buccal means the surface toward the cheek. Lingual is the surface toward the tongue. Palatal is towards the roof of the mouth.

Refs: Bassert, Beal and Samples, McCurnin’s Clinical Textbook for Veterinary Technicians, 10th ed., p. 1118-20.

How well did you know this?
1
Not at all
2
3
4
5
Perfectly
20
Q

Which one of the following phrases is the correct explanation of polychromasia?
An abnormal white blood cell count
Changes in shape of the nuclei of red blood cells
An abnormal red blood cell count
Variable color intensity of red blood cells
The presence of parasites inside the red blood cells

A

Answer: An abnormal red blood cell count.

Polychromasia is present when the color of red blood cells on a blood smear varies, usually from increased numbers of cells with more or less color than usual.

The highly colored cells are immature RBCs released into circulation too early; the color is from cellular organelles that are still present.

A small amount of polychromasia is not abnormal but large numbers means that immature red blood cells are present in circulation.

In anemic animals, this indicates that a regenerative response is present. Polychromatic cells can be seen best on a smear stained with new methylene blue.

Click here to see an image of polychromasia at Cornell’s eClinPath website.
https://eclinpath.com/hematology/morphologic-features/red-blood-cells/color/

Refs: McCurnin and Basset’s, Clin Textbook for Vet Technicians, 9th ed. pp. 380.

How well did you know this?
1
Not at all
2
3
4
5
Perfectly
21
Q

A horse is presented in severe respiratory distress in southern California after a suspected rattlesnake bite. There is significant nasal swelling, flared nostrils, abdominal movement with each breath, stridor, and tachypnea.
The veterinarian is concerned about local tissue damage, upper airway obstruction, and coagulopathy.
In addition to routine CBC and a blood chemistry panel, what tests will be ordered to assess for coagulation problems?

Prothrombin time, partial thromboplastin time
Baseline cortisol levels
Refractometry to estimate plasma protein, including fibrinogen
Lipase and amylase assays
Serum amyloid A acute phase protein assay

A

Answer: Prothrombin time, partial thromboplastin time

A coagulation panel should be run, which generally includes these standard tests for coagulation: prothrombin time (PT), activated partial thromboplastin time (aPTT), and sometimes activated clotting time (ACT).
Coagulopathies are common in association with pit viper (crotalia; e.g., rattlesnake) envenomation. Additional concerns include thrombocytopenia, neurotoxicity, and cardiovascular toxicity.
In contrast, expect hemolysis, lower motor neuron paralysis, and mental depression with coral snake (Micrurus spp.) envenomation.
Clinical signs with pit viper envenomation typically develop rapidly (w/in 15 minutes), whereas onset of clinical signs with coral snake bites can be delayed up to 36 hours (though usually progress extremely rapidly (within minutes)).

How well did you know this?
1
Not at all
2
3
4
5
Perfectly
22
Q

A two-year-old female spayed great Dane dog is presented with a one-month history of focal alopecia, scaling, and crusting around the ears and face.
Cytology of a skin scraping is shown below.
What is the most effective systemic treatment?

Praziquantel
Lufenuron and milbemycin oxime
Amitraz and ivermectin
Amphotericin-B
Terbinafine

A

Answer: Terbinafine

Terbinafine is the drug of choice for canine dermatophytosis (ringworm). One treatment regimen is 30-40 mg/kg PO q 24h or week on/week off until cured.
Itraconazole is the drug of choice for cats, but may be cost-prohibitive in dogs. Amphotericin-B is a systemic antifungal, but is not used to treat ringworm.
Dermatophytosis is a fungal infection of keratinized tissue (skin, hair, and claws), usually caused by Microsporum canis in cats and dogs. The incidence of disease is higher in cats than dogs and it is most common in young animals.
Look for focal/multifocal, sometimes circular, regions of alopecia, scaling, and crusting with variable inflammation and pruritus.
Confirm Dx with fungal culture of hair and skin scale.
Tx can be local or systemic and is generally recommended to minimize spread and shorten the clinical course of disease.
Dermatophytosis is a zoonosis (so wash your hands or wear gloves)!

CGPT: Dermatophytosis in Dogs and Cats

How well did you know this?
1
Not at all
2
3
4
5
Perfectly
23
Q

In addition to daily insulin injections, which interventions are important for treatment of non-ketotic diabetes mellitus in dogs (and cats)?

Glipizide; restrict activity; urinary acidifiers
Antidiuretic hormone; Florinef®; fluid restriction
Glucotrol®; high calorie/low fiber diet; promote water consumption
Balanced diet; exercise; treat concurrent diseases
Omeprazole; low sodium/highly digestible diet;
free choice water

A

Answer: Balanced diet; exercise; treat concurrent diseases

Treat uncomplicated diabetes mellitus (DM) with insulin, a balanced diabetic-appropriate diet (typically fed q12h), exercise (decreases insulin requirement), and treatment of concurrent diseases.
Diet formulation depends on an animal’s body condition, species, and concurrent diseases. In dogs, consider a diet higher in complex carbohydrates (i.e., fiber). In cats, consider a low-simple carbohydrate diet.
Note - glipizide and Glucotrol® are the same oral hypoglycemic drug, no longer recommended for the treatment of DM in cats.

CGPT Comprehensive Summary on the Treatment of Diabetes Mellitus (DM) in Dogs and Cats

How well did you know this?
1
Not at all
2
3
4
5
Perfectly
24
Q

What does it mean if a cat has physiologic nystagmus?

Head shaking related to inner ear disturbance
Abnormally loud gastrointestinal sounds in the absence of a recent meal
Genital malformation
Loss of balance
Normal, rapid, involuntary movement of eyes as head moves side to side

A

Answer: Normal, rapid, involuntary movement of eyes as head moves side to side

Physiologic nystagmus is a normal, rapid, involuntary movement of both eyes that occurs when an animal’s head is moved from side to side.
You can see it easily if you try it on your own dog or cat. Click here to see a short demonstration video of nystagmus (human eye).
Nystagmus can also be abnormal due to disease processes that may cause nystagmus when the head is stationary, or make the eyes move in abnormal directions as the head moves.
Vestibular disease can be peripheral (e.g., otitis media and otitis interna or nasopharyngeal polyps) or central (e.g., FIP, toxoplasmosis, neoplasia).

CGPT: Comprehensive Summary on Feline Infectious Peritonitis (FIP)

CGPT: Comprehensive Summary on Toxoplasmosis in Animals

How well did you know this?
1
Not at all
2
3
4
5
Perfectly
25
Q

A mature goat presents with drooling, left-sided ptosis, a droopy left ear, and positional nystagmus. The goat is also febrile, lethargic, and circling to the left.
Which one of the following choices is the best treatment for the presumptive diagnosis?

Ponazuril (PO, 28 days), sulfadiazine with pyrimethamine
Thiamine (TID, IV 1st dose, then IM), dexamethasone and supportive care
Clarithromycin (PO, BID) and rifampin
High dose oxytetracycline or penicillin
Specific botulinum antitoxin, NSAIDs, and B-lactam antibiotics

A

Answer: High dose oxytetracycline or penicillin

Treat this likely case of listeriosis with high doses of tetracyclines or penicillin. Isolate the goat and take it off silage, +/- IV fluids.
Some of the signs are subtle in this image, but classically look for unilateral ipsilateral signs of brainstem and facial nerve dysfunction
-swelling of cheek due to accumulation of food in the buccal space, drooling, ptosis (lowered eyelid), drooping ear, collapsed nostril, flaccid lip.
Typically listeriosis presents as encephalitis (depressed, fever) with unilateral limb signs (circling, leaning) and unilateral cranial nerve signs as discussed above. Also can see sporadic ABORTIONS with listeriosis at any time in gestation, though more common in 3rd trimester.
Listeria is ZOONOTIC. Highest risk is handling aborted fetus or performing necropsy of septicemic animal.
With polioencephalomalacia, think opisthotonos, headpressing, blind, ‘star-gazing’ (dorso-medial strabismus via thiamine deficiency).
Caprine arthritis encephalitis (CAE) presents as a polyarthritis in adults, progressive paresis in kids.

Key Information on Listeriosis in Animals
Definitions
• Listeriosis: A bacterial infection caused by Listeria monocytogenes, leading to encephalitis, abortion, septicemia.

Causative Agent
• Organism: Listeria monocytogenes, a gram-positive, motile, nonspore-forming coccobacillus.

Epidemiology
• Reservoirs: Soil, GI tracts of mammals, silage.
• Transmission: Ingestion of contaminated feed, especially spoiled silage; fecal-oral route.

Clinical Findings
• Ruminants: Encephalitis, asymmetric brain-stem dysfunction (circling, head tilt, facial paralysis), abortion, septicemia.
• Other Animals: Septicemia in monogastric animals, abortion in all domestic animals.

Diagnosis
• Clinical Signs: Asymmetric neurologic signs, abortion without prior symptoms.
• Tests: CSF analysis (increased protein, mild pleocytosis), bacterial culture, PCR, immunofluorescence.

Treatment
• Antimicrobials: High doses of penicillin G, oxytetracycline, ceftiofur, erythromycin.
• Supportive Care: Fluids, electrolytes, corticosteroids.

Control and Prevention
• Silage Management: Avoid feeding spoiled silage, use acidic silage additives.
• Vaccination: Efficacy varies; not always recommended.

Zoonotic Risk
• Human Infection: Handling of contaminated materials; can cause severe illness in immunocompromised individuals.

CGPT: Comprehensive Summary on
Polioencephalomalacia (PEM) in Ruminants

CGPT: Comprehensive Summary on Caprine Arthritis and Encephalitis (CAE)

How well did you know this?
1
Not at all
2
3
4
5
Perfectly
26
Q

Which of the following choices is the correct description of guttural pouch tympany?

Viral infection of the guttural pouch
Bacterial infection of the guttural pouch
Air trapped in the guttural pouch
Fungal infection of the guttural pouch
Fracture of the stylohyoid bone within the guttural pouch

A

Answer: Air trapped in the guttural pouch

Guttural pouch tympany is trapping of air in the guttural pouch typically seen in foals. Look for a non-painful fluctuant swelling on one or both sides of the throat.
Guttural pouch empyema is a bacterial infection of the guttural pouch. It is most often seen with or following strangles infection.
Guttural pouch mycosis is a fungal infection of the guttural pouch.
Epistaxis is the most common clinical sign.
Fracture of the stylohyoid bone within the guttural pouch can be part of a disorder called temporohyoid osteoarthropathy.

Comprehensive Summary on Guttural Pouch Tympany in Horses

Definition and Etiology
Guttural Pouch Tympany (GPT) is a condition characterized by the distension of one or both guttural pouches with air. It primarily affects young horses from birth to 1.5 years of age and is more common in fillies than in colts. The exact cause is often unknown, but it may involve a congenital defect or inflammation affecting the mucosal flap at the pharyngeal orifice.

Pathophysiology
• Abnormal Air Accumulation: Dysfunction or malformation of the plica salpingopharyngea creates a one-way valve effect, allowing air to enter the pouch but preventing its escape, leading to tympany.
• Potential Secondary Causes: In some cases, upper respiratory tract inflammation might contribute to the dysfunction.

Clinical Signs
• Characteristic Swelling: Nonpainful, fluctuant swelling in the parotid region (behind the jaw).
• Unilateral/Bilateral Tympany: Usually unilateral, but bilateral cases can occur.
• Respiratory Signs: Stertorous breathing, nasal discharge, and head carriage extension in severe cases.
• Complications: Dysphagia, dyspnea, secondary empyema, and aspiration pneumonia in complicated cases.

Diagnosis
• Clinical Signs: Based on observable swelling and respiratory signs.
• Radiographic Examination: X-rays of the skull to confirm the presence of air in the guttural pouch.
• Endoscopy: To confirm diagnosis, rule out other conditions, and distinguish between unilateral and bilateral disease.

Treatment
1. Conservative Management:
• NSAIDs: Nonsteroidal anti-inflammatory drugs to reduce inflammation.
• Antibiotics: Broad-spectrum antimicrobials if secondary infection is suspected.
2. Decompression:
• Percutaneous Needle/Catheter: Temporary relief by releasing air via a needle or catheter.
• Surgical Intervention:
• Fenestration: Creating a window between the affected and normal pouch to allow air escape.
• Foley Catheters: Long-term placement to remodel the mucosal flap and prevent air trapping.

Prognosis
• Good Prognosis: Most cases respond well to treatment, particularly if there are no secondary complications.
• Surgical Cases: May require repeated procedures, especially for bilateral tympany.

How well did you know this?
1
Not at all
2
3
4
5
Perfectly
27
Q

A nine-year-old female spayed cat is presented for polydipsia, polyuria and weight loss in spite of a voracious appetite. The cat has an unkempt coat and a rapid heart rate. The thyroid is palpably bilaterally enlarged.
What is the most common cause of feline hyperthyroidism?

Pituitary hyperplasia
Thyroid carcinoma
Hypercalcemic secondary parathyroidism
Functional thyroid adenoma
latrogenic

A

Answer: Functional thyroid adenoma

The most common cause of feline hyperthyroidism is a functional thyroid adenoma (adenomatous hyperplasia). In roughly 70% of cases, both thyroid lobes are affected.
Thyroid carcinoma is the primary cause of hyperthyroidism in dogs, but it is rare in cats (1-2% of hyperthyroidism cases).

Comprehensive Summary on Hyperthyroidism in Animals for BCSE Test Preparation

Definition and Etiology

Hyperthyroidism is a condition caused by excessive thyroid hormone production, primarily thyroxine (T4) and triiodothyronine (T3). In cats, it is usually due to thyroid adenomas or adenomatous hyperplasia. Thyroid carcinoma is a rare cause in cats but more common in dogs.

Pathophysiology

•	Excessive Hormone Production: Leads to increased metabolic rate.
•	Adenomatous Hyperplasia: Most common in cats, leading to bilateral thyroid enlargement.
•	Thyroid Carcinoma: Common in dogs, causing similar clinical signs.

Clinical Signs

•	Common Signs in Cats:
•	Hyperactivity, excessive vocalization, weight loss despite good appetite.
•	Vomiting, polyuria, polydipsia, tachycardia, and possible thyroid nodule.
•	In Dogs: Often associated with thyroid carcinoma; signs include weight loss, increased appetite, and possible palpable thyroid mass.

Diagnosis

•	Clinical Signs and History: Key to initial suspicion.
•	Serum Thyroid Hormone Testing: Elevated total T4 or free T4 (fT4) concentrations confirm diagnosis.
•	Additional Tests: CBC, serum biochemistry, urinalysis to assess overall health and rule out concurrent diseases.

Treatment

1.	Radioiodine Therapy: Preferred curative treatment; selectively destroys hyperactive thyroid tissue.
2.	Antithyroid Medications:
•	Methimazole: Initial dose of 1.25–2.5 mg/cat every 12 hours, adjusted based on T4 levels.
•	Carbimazole: Similar to methimazole, converted to methimazole in the body.
•	Propylthiouracil: Not recommended due to severe side effects.
3.	Surgical Thyroidectomy: Option for definitive treatment, especially in cases not suited for radioiodine therapy.
4.	Iodine-Deficient Diet: Lifelong dietary management to limit iodine intake and reduce hormone production.

Medications

•	Methimazole: Blocks thyroid hormone synthesis; dose adjustment based on response and side effects.
•	Carbimazole: Prodrug converted to methimazole; used where methimazole is tolerated.
•	Beta-Adrenoceptor Blockers (Propranolol, Atenolol): Used to control tachycardia and hypertension but do not affect thyroid hormone levels.

Monitoring and Follow-Up

•	Regular Monitoring: Essential to adjust treatment and monitor for side effects or development of hypothyroidism.
•	Renal Function: Monitor as hyperthyroidism can mask underlying kidney disease.
How well did you know this?
1
Not at all
2
3
4
5
Perfectly
28
Q

Immediately after breeding, a three-year-old male poodle is unable to retract his penis back into the prepuce.
On exam, the penis is cyanotic and engorged and the dog is uncomfortable. The skin surrounding the preputial orifice is inverted, constricting the penis. No other abnormalities are observed.
What is the best treatment?

Do nothing; penile swelling will resolve and the penis will retract into the prepuce naturally

Gently cleanse and lubricate the penis; attempt to replace the penis into the prepuce

Administer general anesthesia; perform partial penectomy and orchiectomy

Administer local anesthesia, place a urinary catheter and perform a perineal urethrostomy

Administer a tranquilizer; surgically enlarge the preputial orifice and replace the penis into the prepuce

A

Answer: Gently cleanse and lubricate the penis; attempt to replace the penis into the prepuce

Paraphimosis is a medical emergency; gently cleanse and lubricate the penis and attempt to replace it into the prepuce.
Recognize and Tx quickly to prevent permanent damage to the penile and urethral tissue.
Tx goals: Reduce swelling, restore normal blood circulation, and replace the penis back into the prepuce. Use cold compresses or topical hypertonic solutions to reduce swelling. Administer sedation/pain medications as indicated.
In case such as this where the dog is presented immediately medical therapy may be successful. However, some cases require incision of the preputial skin to examine the preputial cavity and relieve venous obstruction.

https://www.merckvetmanual.com/reproductive-system/reproductive-diseases-of-the-male-small-animal/paraphimosis-in-dogs-and-cats

How well did you know this?
1
Not at all
2
3
4
5
Perfectly
29
Q

What part keeps exhaled gasses moving away from the patient in a circular rebreathing anesthetic delivery system?

One-way valves
Rebreathing bag
Oxygen flush valve
Y-valve on endotracheal tube
Passive scavenger

A

Answer: One-way valves

One-way valves help direct gas flow. Expired gasses pass through a CO2 scavenger canister to remove CO2.

STRATEGY HINT: See how the word “valve” occurs in 3 choices? Chances are that the correct answer contains this word.

How well did you know this?
1
Not at all
2
3
4
5
Perfectly
30
Q

To what approximate value should the anesthetic dial initially be set, in order to help most patients reach a surgical plane of anesthesia with an inhalant anesthetic agent such as
isoflurane? (Note: MAC = minimum alveolar concentration)
0.4 × MAC
2.0 x MAC
3.0 × MAC
Equal to MAC
1.5 × MAC

A

Answer: 1.5 x MAC

For most anesthetics and most patients, utilize a dial setting of approximately 1.5 x MAC as a guideline to help the patient reach a surgical plane of anesthesia. Adjust as needed based on how the patient is responding.
The minimum alveolar concentration (MAC) of an anesthetic agent is the percent concentration at which 50% of patients would not respond to surgical stimulation.
1.5 x MAC is a guideline only: assess the patient and use the appropriate MAC for the situation depending on the patient’s condition and pre-operative medications. It is critical to closely monitor patients under anesthesia and dose the anesthetic “to effect.”

How well did you know this?
1
Not at all
2
3
4
5
Perfectly
31
Q

What nerve or nerves need to be blocked in order to dehorn cattle?
Auriculopalpebral
Cornual and infratrochlear
Infratrochlear
Auriculopalpebral and infraorbital
Cornual

A

Answer: Cornual.

Typically a cornual nerve block (branch of the trigeminal- midway between eye and base of horn, just below temporal line) will anesthetize most cows for dehorning.
If that is not enough, do a ring block around the base of the horn.
In contrast, for goats you need to block BOTH the cornual nerve (caudal ridge, root of zygomatic arch of frontal bone) and infratrochlear nerve (dorsomedial to eye, close to edge of bony orbit) before dehorning.
This is because the goat horns lie more rostral on the skull and closer to the bony orbits than the horns of a cow.

How well did you know this?
1
Not at all
2
3
4
5
Perfectly
32
Q

A four-year-old Andalusian gelding is presented three days post-routine open castration performed in the field. The owner is concerned about the amount of swelling in the inguinal region.
The horse is eating, urinating, and defecating normally. Physical exam parameters are within normal limits.
The scrotal region and prepuce have moderate swelling with no drainage from the incisions.
What is the appropriate course of action?

Aseptic aspiration of this hydrocele, tell owner if it recurs surgery is indicated

Open and stretch the incisions, systemic NSAIDs, increase exercise

Identify hemorrhage source, place crushing forceps, pack incision w/ gauze

Ultrasound to confirm inguinal hernia, place support bandage, refer immediately for surgery

Institute systemic antimicrobials and anti-inflammatories, limit horse’s movement

A

Answer: Open and stretch the incisions, systemic NSAIDs, increase exercise.

This horse has routine postoperative swelling and the appropriate course of therapy is to open and stretch the incisions using sterile technique, administer systemic NSAIDs, and increase exercise to promote drainage.
Scrotal and preputial swelling and edema that peak at day 3-4 and resolve by day 10-12 are the most common complication following castration. In the absence of fever or significant pain/reluctance to move or urinate, the therapy noted above can be performed on the farm.
Between 20-38% of geldings experience a complication post-operatively. The 2nd-most common is hemorrhage.
The most serious is eventration of intestine through the incision. Additional complications include peritonitis, penile damage, and hydrocele (occurs months to years later).
Click here for an excellent article on the complications of castration, courtesy of Dr. Liberty Getman and the AAEP.

Intestinal Eventration after castration refers to the prolapse of intestinal contents through the surgical site. This condition is an emergency and is more likely in draft breeds, Tennessee Walking Horses, American Saddlebreds, Standardbreds, and adult stallions due to their larger inguinal rings.

Pathophysiology
• Post-Surgical Complication: Typically occurs within 4 hours to 6 days post-castration.
• Herniation Mechanism: Intestinal loops or omentum protrude through the inguinal canal.

Clinical Signs
• Visible Prolapse: Intestine or omentum hanging from the surgical site.
• Acute Pain: Due to stretching and possible strangulation of the intestine.

Diagnosis
• Physical Examination: Identification of protruding structures.
• Transrectal Palpation: To assess the extent of the prolapse.

Treatment
1. Omental Evisceration:
• Restraint and Sedation: Ensures the horse remains calm.
• Emasculation: Removal of the omentum if involved.
• Scrotal Packing and Closure: Sterile gauze and suturing.
• Antibiotics and NSAIDs: Prevent infection and manage pain.
2. Intestinal Evisceration:
• General Anesthesia: For safe examination and treatment.
• Intestinal Lavage: Clean and assess the intestine.
• Reduction or Surgical Referral: Depending on the damage, intestine may be reduced or referred for surgical resection.

Medications
• Broad-Spectrum Antibiotics: To prevent or treat infections.
• NSAIDs: For pain and inflammation control.
• Tetanus Prophylaxis: To prevent tetanus infection.

How well did you know this?
1
Not at all
2
3
4
5
Perfectly
33
Q

Which part of the stomach and foregut system occupies the most space in a newborn calf?
Abomasum
Omasum
Rumen
Reticulum
Torus pyloricus

A

Answer: Abomasum

The abomasum takes up more 50% of the stomach and foregut system in the newborn calf. As the calf is weaned from an all milk diet to roughage, the abomasum shrinks, and the rumenoreticulum system becomes active.

34
Q

A four-year-old Doberman pinscher mix is presented after being hit by a car. The dog is non-weight bearing and knuckling on the right thoracic limb and has ipsilateral Horner syndrome.
There is a brachial plexus avulsion.
How does a lesion at C6-T cause Horner syndrome in the right eye?

Damage to cervical sympathetic innervation of the eye
Interrupts inhibition of the oculomotor nucleus
Damage to typanooccipital postsynaptic ganglion
Damage to cervical parasympathetic innervation of the eye
Interrupts nerve transmission via caudal cervical ganglion

A

Answer: Damage to cervical sympathetic innervation of the eye.

Horner syndrome secondary to brachial plexus avulsion occurs due to damage to the cervical sympathetic innervation to the eye, which originates at T1 - T2 and courses cranially in the sympathetic trunk of the cervical spinal cord.
Clinical signs vary depending on lesion severity. Complete spinal nerve avulsion causes flaccid paralysis of the limb, dropped elbow, knuckling, and loss of sensation distal to the elbow. Look also for ipsilateral loss of the cutaneous trunci reflex and ipsilateral Horner syndrome.
Tx: Consider limb amputation for complete brachial plexus avulsion to prevent ongoing trauma to the distal limb. In some cases, in which only inflammation of the plexus occurred,
clinical signs improve with time.

Comprehensive Summary on Intestinal Eventration After Castration for BCSE Test Preparation

Definition and Etiology
Intestinal Eventration after castration refers to the prolapse of intestinal contents through the surgical site. This condition is an emergency and is more likely in draft breeds, Tennessee Walking Horses, American Saddlebreds, Standardbreds, and adult stallions due to their larger inguinal rings.

Pathophysiology
• Post-Surgical Complication: Typically occurs within 4 hours to 6 days post-castration.
• Herniation Mechanism: Intestinal loops or omentum protrude through the inguinal canal.

Clinical Signs
• Visible Prolapse: Intestine or omentum hanging from the surgical site.
• Acute Pain: Due to stretching and possible strangulation of the intestine.

Diagnosis
• Physical Examination: Identification of protruding structures.
• Transrectal Palpation: To assess the extent of the prolapse.

Treatment
1. Omental Evisceration:
• Restraint and Sedation: Ensures the horse remains calm.
• Emasculation: Removal of the omentum if involved.
• Scrotal Packing and Closure: Sterile gauze and suturing.
• Antibiotics and NSAIDs: Prevent infection and manage pain.
2. Intestinal Evisceration:
• General Anesthesia: For safe examination and treatment.
• Intestinal Lavage: Clean and assess the intestine.
• Reduction or Surgical Referral: Depending on the damage, intestine may be reduced or referred for surgical resection.

Medications
• Broad-Spectrum Antibiotics: To prevent or treat infections.
• NSAIDs: For pain and inflammation control.
• Tetanus Prophylaxis: To prevent tetanus infection.

35
Q

A horse is presented with an acute wound over the dorsal aspect of the mid-metatarsal region of the right hind limb.
The horse is walking fairly well but seems unable to completely protract the limb and occasionally knuckles over on the fetlock.
Which of the following tendon or ligament pairs is likely damaged?
Impar ligament and long plantar ligament
Peroneus tertius muscle and gastrocnemius tendon
Cunean tendon and lateral collateral ligament
Long and lateral digital extensor tendons
Deep and superficial digital flexor tendons

A

Answer: Long and lateral digital extensor tendons

The long and lateral digital extensor tendons have likely been disrupted.
Generally there is minimal affect of extensor tendon injuries on gait; however, in acute phases or with severe injuries may see inability to protract the limb and intermittent knuckling over on the fetlock.
Locomotion typically returns to normal after healing of the wound occurs. In some cases trauma to the lateral digital extensor tendon may be associated with the development of stringhalt (exaggerated flexion of the limb at a walk).

Comprehensive Summary on Long and Lateral Digital Extensor Tendons for BCSE Test Preparation

Definition and Function
• Long Digital Extensor Tendon (LDET): Extends the digits and stabilizes the stifle joint.
• Lateral Digital Extensor Tendon (LDE): Assists in extending the digits and stabilizing the lower limb.

Injury and Pathophysiology
• Injuries: Commonly occur due to trauma, such as lacerations or overstretching.
• Symptoms: Inability to extend the digit properly, resulting in a dropped fetlock or knuckling over of the limb.

Clinical Signs
• Lameness: Varying degrees depending on the severity of the injury.
• Swelling and Heat: Around the affected tendon.

Diagnosis
• Physical Examination: Observation of the limb’s position and movement.
• Ultrasound: To assess the extent of the damage to the tendons.
• Radiography: To rule out concurrent fractures.

Treatment
1. Initial Management:
• Wound Care: Clean and debride any open wounds.
• Stabilization: Bandaging and splinting to prevent further damage.
2. Medical Treatment:
• NSAIDs: For pain relief and to reduce inflammation.
• Antibiotics: If there’s an open wound or risk of infection.
3. Surgical Intervention:
• Repair: Suturing the tendon if necessary.
• Rehabilitation: Gradual reintroduction to activity with controlled exercise.

Prognosis
• Depends on Severity: Early intervention and appropriate management significantly improve outcomes.

36
Q

A 12-year-old intact female cat is presented for a firm, ulcerated, 2x2 cm mass in the right middle mammary gland. Thoracic radiographs do not show evidence of metastasis and minimum database of bloodwork is within normal limits.
The plan is to perform a unilateral mastectomy. Ligate which arteries during this procedure?

Mesenteric and long internal pudendal
Medial circumflex and external obturator
Ventral perineal and cranial pancreaticoduodenal
Cranial and caudal superficial epigastric
Internal iliac and right gastroepiploic

A

Answer: Cranial and caudal superficial epigastric

Ligate the cranial and caudal superficial epigastric arteries, the primary blood supply to the mammary glands, during a unilateral mastectomy.
For the caudal chain of glands, ligate the caudal superficial epigastric arteries emerging from the external pudendal artery near the inguinal canal. For the cranial chain, ligate the cranial superficial epigastric artery, arising from the internal pudendal artery. The lateral thoracic artery from the axillary artery also supplies the cranial-most mammary gland.
Cats usually have four pairs of glands while dogs have five.
Mammary gland neoplasia is most common in older intact females; more common in dogs than in cats, but 90% of feline tumors are malignant vs. 45% in dogs.

37
Q

Where is a preliminary tuberculosis (TB) screening test performed on a cow?
Left ear base
Cervical skin
Ventral udder skin
Caudal tail fold
Caudo-dorsal to eye

A

Answer: Caudal tail fold.

Use the intradermal caudal (tail) fold test (CFT) as a preliminary screening test for bovine tuberculosis.
The confirmatory test is the comparative cervical test (CCT), done on the neck skin only AFTER a positive CFT.
TB is an granulomatous disease caused by Mycobacterium spp. that can cause acute or chronic, debilitating Dz. It is of huge public health and financial concern because it is zoonotic and there are multidrug resistant strains emerging. Test and slaughter is the most common method of handling Dz in North America.

The tail fold test is used for diagnosing tuberculosis in cattle, particularly in countries with high TB prevalence. It involves the intradermal injection of a small amount of tuberculin into the skin at the base of the tail. After 72 hours, the injection site is examined for a delayed-type hypersensitivity reaction, such as swelling or thickening of the skin. A positive reaction indicates possible TB infection, necessitating further diagnostic testing and potential culling to prevent disease spread.

38
Q

Which of the following choices best describes phimosis?
Failure to retract the penis into the preputial cavity after intromission
Inflammation of the penile and preputial mucosa
Persistent abnormal erection of the penis
Inability to extrude penis due to constriction of the preputial orifice
Incapable of achieving an erection

A

Answer: Inability to extrude penis due to constrictions of the preputial orifice

Phimosis is an inability to extrude the penis due to constriction at the preputial orifice. It may be congenital (German shepherd, golden retriever) or acquired (trauma, post-surgical).
In severe cases urine accumulation within the prepuce results in urine dribbling and balanoposthitis. Tx with surgery to enlarge the preputial opening.
Paraphimosis is the inability to retract the penis into the preputial cavity.
Balanoposthitis is inflammation of both the penile (balanitis) and preputial (posthitis) mucosa.
Priapism is a persistent abnormal erection of the penis in the absence of sexual stimulation, generally accompanied by pain and sensitivity to touch.

CGPT: Phimosis in Dogs and Cats

CGPT Praphimosis in Dogs and Cats

https://www.merckvetmanual.com/reproductive-system/reproductive-diseases-of-the-male-small-animal/balanoposthitis-in-dogs-and-cats

39
Q

A dog is presented for thoracolumbar pain, hindlimb hypermetria, and ataxia.
On neurologic exam, the cutaneous trunci reflex is present until reaching a line adjacent to the level of the first lumbar vertebra (L1).
Which of the following is the most likely location of the lesion?
C7-T1
L6-L7
L3-4
T12-T13
T1-T2

A

Answer: T12-T13

The dog’s lesion is most likely at T12-13. The loss of the cutaneous trunci reflex occurs about 1-4 (usually 2) spinal cord segments CAUDAL to the actual lesion.
Another way to think of this is that the actual spinal lesion is generally 2 spinal cord segments in FRONT (cranial) to the line where the cutaneous trunci reflex STOPS.
Dorsal cutaneous branches do not exit the spinal cord at around L3-4 so the the loss of the reflex on the left at L3 is normal.
The lateral thoracic nerve, which innervates the ipsilateral cutaneous trunci muscle, exits the spinal cord at C7-1. So with a brachial plexus avulsion you may see an ipsilateral loss of the cutaneous trunci twitching while lightly pinching on the skin on the ipsilateral side, however the contralateral side will twitch!

Cutaneous Trunci Reflex in Veterinary Neurology

Definition

The cutaneous trunci reflex is a test to assess the integrity of the spinal cord, particularly the thoracolumbar region.

Procedure
• Method: Gently squeeze the skin lateral to the spine with a hemostat, starting from the lumbosacral region and moving cranially.
• Normal Response: Bilateral contraction of the cutaneous trunci muscles, resulting in a skin twitch over the thorax and abdomen.

Clinical Significance
• Lesion Localization: Absence of reflex caudal to a lesion, while the reflex remains normal cranial to it.
• LMN Involvement: Indicates involvement of the lateral thoracic nerve (C8-T2 spinal segments).

Application
• Use: Useful for diagnosing spinal cord injuries and differentiating them from peripheral nerve injuries.

40
Q

What is the function of the objects highlighted by the arrows below?

Remove red blood cell parasites
Engulf bacteria
Prevent phagocytosis
Help blood clot
Transport immunoglobulins

A

Answer: Help blood clot

These are thrombocytes (platelets), which help blood clot.
Formation of the platelet plug is the first step in hemostasis (“primary hemostasis”).
Platelets are also critical in formation of the fibrin plug (“secondary hemostasis”).
Follow these links to see clumps of bovine platelets and scattered feline platelets (and rouleaux formation - i.e., stacked up erythrocytes).
Res: McCurnin & Bassert, Clin Textbook for Vet Technicians, 9th ed. p. 384. Image courtesy of Dr. Seth Chapman.

Platelets in Animals - Key Points for Veterinary Professionals
Definition and Function
• Platelets: Cell fragments derived from megakaryocytes in the bone marrow.
• Role: Form initial hemostatic plug during hemorrhage, source of phospholipid for coagulation factors to form fibrin clots.

Physiopathology
• Production: Influenced by thrombopoietin, involves megakaryocyte membrane invagination and cytoplasmic fragmentation.
• Structure: Dense granules (ATP, ADP, calcium), serotonin, lysosomes, glycogen, mitochondria, intracellular canalicular system.

Hemostasis Process
1. Vessel Damage: Exposes collagen and tissue factor.
2. Platelet Adhesion: Via von Willebrand factor.
3. Shape Change: Release of ADP.
4. Aggregation: Forms primary platelet plug.
5. Fibrin Clot Formation: Consolidated by platelet contractile proteins.

Disorders
• Thrombocytopenia: Low platelet count (<30,000/μL), increased hemorrhage risk.
• Causes: Consumption (hemorrhage, DIC), destruction (immune-mediated), sequestration (hypersplenism).
• Symptoms: Petechiae, ecchymoses, mucosal bleeding.
• Thrombocytosis: Rare, often idiopathic or related to marrow disease.
• Thrombocytopathies: Normal count, impaired function (e.g., von Willebrand disease, aspirin-induced thromboxane inhibition).

Medications and Management
• Treatment for Thrombocytopenia: Address underlying cause, immunosuppressants for immune-mediated cases.
• Thrombocytosis: Address primary condition, if identified.
• Platelet Function Disorders: Avoid causative drugs, supportive care.

41
Q

Hemoglobin concentration is roughly what percentage of packed cell volume (PCV) in most domestic animals?
50% of PCV
10% of PCV
33% of PCV
25% of PCV

A

Answer: 33%

Normal hemoglobin concentration (Hb, g/dl) is about one-third (33%) of the packed cell volume (PCV) in normal dogs, cats, horses, cows, sheep, goats, and pigs.
So, in a cat with a PCV of 30%, the Hb will be roughly 10 g/dl.
And in a dog with a PCV of 45%, the Hb will be roughly 15 g/dl.
Remember, Hb is a protein in erythrocytes that helps to transport oxygen.
Follow this link to see Small and large Animal reference values.
One exception to the 33% rule is South American camelids, whose hemoglobin is 40%-45% of the PCV, presumably an adaptation to high altitude, lower oxygen environments.

CGPT: Hemoglobin Test in Veterinary Hematology

https://www.merckvetmanual.com/special-subjects/reference-guides

42
Q

An osteoclast is a specialized type of which cell?
Macrophage
Mast cell
Band neutrophil
Glial cell
Basophil

A

Answer: Macrophage

Osteoclasts are macrophages that are associated with bone.
When circulating monocytes become associated with a tissue they are called resident or tissue macrophages and are often given a new name.
Some of the other key tissue-associated resident macrophages are:
Kupffer cells, associated with the liver.
Histiocytes, associated with connective tissue.
Microglial cells, associated with nervous tissue.
Pulmonary alveolar macrophages of the lungs.
Sinusoidal lining cells of the spleen.

43
Q

A four-year-old female spayed cocker spaniel is presented with a one-day history of bleeding from the gums.
Exam reveals diffuse petechiation of the mucous membranes. There is hyphema OS and ecchymosis and petechiation on the inner pinnae.

What is the most likely diagnosis?
Hepatic insufficiency
Immune-mediated thrombocytopenia
Von Willebrand disease
Anticoagulant rodenticide toxicity
Disseminated intravascular coagulation (DIC)

A

Answer: Immune - mediated thrombocytopenia

This is immune-mediated thrombocytopenia (a.k.a. ITP), the most common cause of profound thrombocytopenia in dogs.
With ITP look for a platelet count <30,000/L (often <10,000/uL) and normal coagulation parameters.
Most commonly idiopathic/primary, but a Dx of exclusion so rule out other causes of thrombocytopenia - e.g., vector-borne Dz, neoplasia.
Tx: Immunosuppression (e.g., prednisone, +/- cyclosporine, or other immunosuppressants). Consider one dose of IV vincristine to stimulate platelet release from the bone marrow.
Dogs may require blood transfusion if significant blood loss. Note: blood transfusion does not replace circulating platelets.

Immune-Mediated Thrombocytopenia (ITP) in Animals

Definition
• ITP: Immune-mediated destruction of platelets or marrow megakaryocytes.
• Also Known As: Idiopathic thrombocytopenic purpura.

Causes and Risk Factors
• Idiopathic: No identifiable underlying cause.
• Risk Factors: Middle-aged, spayed female dogs, especially Cocker Spaniels.

Pathophysiology
• Mechanism: Immune system targets and destroys platelets or megakaryocytes.
• Autoantibodies: Directed against platelet surface antigens.

Clinical Signs
• Bleeding: Petechiae, ecchymoses, melena, epistaxis.
• Severe Thrombocytopenia: Platelet counts <30,000/μL, often <10,000/μL.

Diagnosis
• Exclusion: Rule out other causes of thrombocytopenia.
• Tests: Bone marrow aspirate (rarely needed), platelet count, clinical signs.

Treatment
• Corticosteroids: High-dose initial treatment, tapered gradually.
• Vincristine: 0.01-0.02 mg/kg IV, shortens recovery time.
• Transfusion: Fresh whole blood for severe anemia.
• Splenectomy: For recurrent cases.
• Monitoring: Regular platelet count checks during steroid tapering.
• Avoid: Drugs interfering with coagulation.

44
Q

Which one of the following physical properties of parasite eggs allows the fecal floatation method to work?
Specific gravity
Air content in the outer shell
Cell wall structure
Weight
Temperature

A

Answer: specific gravity

Specific gravity compares the density of a substance to that of water.
Substances with a specific gravity less than water will float.
The fecal flotation test relies on the fact that the specific gravity of the flotation solution used is greater than that of the parasite eggs or oocysts.
The eggs or oocysts will float and can be found more easily.
A fecal sample is mixed in a vial with a concentrated solution of a salt or sugar in water.
The vial is covered with a coverslip and then left undisturbed.
The eggs will float to the top and are examined using the low power objective of a microscope.

45
Q

Salmonella typhimurium is associated with what condition in pigs?
Rectal stricture
Intussusception
Swine dysentery
Rectal prolapse
Secondary to intestinal spirochetosis

A

Answer: Rectal stricture.

Salmonella typhimurium is associated with rectal strictures in growing pigs. Caused by an ulcerative proctitis that damages rectal tissue.

Can see large numbers of cases. Can see sporadic rectal strictures as a sequelae to rectal prolapse.

Intestinal spirochetosis is a post-weaning diarrhea seen in the ABSENCE of Brachyspira (Serpulina) hyodysenteriae (swine dysentery), but similar in presentation to it. This syndrome is being recognized more frequently worldwide.

Refs: Ogilve, LA Int Med, pp. 80-4.

Link: Rectal Strictures in Pigs
https://www.merckvetmanual.com/digestive-system/intestinal-diseases-in-pigs/rectal-strictures-in-pigs

Link: Rectal Prolapse in Animals
https://www.merckvetmanual.com/digestive-system/diseases-of-the-rectum-and-anus/rectal-prolapse-in-animals?redirectid=31728

Link: Intestinal Spirochetosis in Pigs
https://www.merckvetmanual.com/digestive-system/intestinal-diseases-in-pigs/intestinal-spirochetosis-in-pigs

Link: Swine Dysentery
https://www.merckvetmanual.com/digestive-system/intestinal-diseases-in-pigs/swine-dysentery

Rectal Strictures in Pigs - Detailed Veterinary Summary
Etiology and Pathogenesis
• Causative Agent: Salmonella enterica serotype Typhimurium
• Pathogenesis: Fibrosis due to persistent ischemia from ulcerative proctitis
Clinical Findings
• Symptoms: Bloating, emaciation, history of severe diarrhea
• Lesions: Narrowed rectal canal, annular fibrotic ulcers, grossly distended colon filled with gas and green feces

Diagnosis
• Physical Examination: Resistance to passing index finger into rectum
• Postmortem Examination: Confirm distended colon and narrowed rectal canal
Treatment and Control
• Management: Good housing, sanitation, all-in/all-out system
• Surgery: Not economically feasible
Prevention
• Control of Diarrhea: Early diagnosis and treatment essential
Key Points
• Strictures result from trauma or infections affecting rectal blood supply
• Effective management and sanitation are crucial
• Diagnosis involves physical and postmortem examination

46
Q

A young horse presents with fever, nasal discharge, and mandibular lymphadenopathy. The horse is isolated from the rest of the herd; a sample of the nasal discharge is PCR positive for Streptococcus equi sbsp. equi. You advise the barn manager to check the temperature twice daily on the other horses. Two days later he calls because one of the other horses has spiked a fever. Which one of the following choices is the best recommendation for this new horse with a fever?

Isolate this horse immediately
Vaccinate this horse against Strep. equisbsp. equi
Start on rifampin orally twice a day
Apply cold packs to submandibular lymph nodes
Call back if it spikes a fever again

A

A young horse presents with fever, nasal discharge, and mandibular lymphadenopathy. The horse is isolated from the rest of the herd; a sample of the nasal discharge is PCR positive for Streptococcus equi sbsp. equi. You advise the barn manager to check the temperature twice daily on the other horses. Two days later he calls because one of the other horses has spiked a fever. Which one of the following choices is the best recommendation for this new horse with a fever?

Isolate this horse immediately
Vaccinate this horse against Strep. equisbsp. equi
Start on rifampin orally twice a day
Apply cold packs to submandibular lymph nodes
Call back if it spikes a fever again

47
Q

Enzootic pneumonia in swine is caused by which one of the following agents?
Pasteurella multocida
Fusobacterium necrophorum
Swine influenza virus
Cytomegalovirus (Herpes)
Mycoplasma hyopneumoniae

A

Answer: Mycoplasma hyopneumoniae

Mycoplasma hyopneumoniae causes enzootic pneumonia in swine.

It is common to see a secondary infection on top of a smoldering mycoplasmal cough with Pasteurella multocida.

Refs: Jackson and Cockcroft, Handbook of Pig Medicine, 1st ed. pp. 71-4.

https://www.merckvetmanual.com/respiratory-system/respiratory-diseases-of-pigs/mycoplasmal-pneumonia-in-pigs

Mycoplasmal Pneumonia in Pigs - Comprehensive Information for BCSE Test
Definitions and Causative Agents:
• Mycoplasmal Pneumonia: Chronic respiratory disease caused by Mycoplasma hyopneumoniae.
• Causative Agents: Mycoplasma hyopneumoniae, often complicated by other mycoplasmas, bacteria, and viruses.

Epidemiology:
• Transmission: Aerosol, direct contact, from dam to piglets.
• Age Susceptibility: Pigs of all ages, highest prevalence in pigs 3-5 months old.
Clinical Findings:
• Signs: Persistent dry cough, reduced growth rate, sporadic severe pneumonia.
• Lesions: Gray or purple, consolidated lung areas, primarily in apical and cardiac lobes, histological lymphoid hyperplasia.

Diagnosis:
• Methods: Clinical signs, histopathology, PCR, serologic tests.
• Samples: Tracheal, laryngeal, bronchial swabs, lung tissue.
Treatment:
• Antibiotics: Effective against Mycoplasma spp (e.g., tetracyclines, macrolides).
• Supportive Care: Improved ventilation, reduced stress.
Control:
• Vaccination: Reduces clinical signs, does not prevent infection.
• Management: All-in/all-out practices, gilt acclimation.

48
Q

What is the normal host of the meningeal worm, Parelaphostrongylus tenuis, which infects camelids?

A

Answer: White-tailed deer

The normal host of P. tenuis is the white-tailed deer (who do not develop clinical signs). Camelids and small ruminants are aberrant hosts.

It is a common disease in camelids, causing high morbidity and mortality.

The larvae of P. tenuis migrate through the spinal cord leading to neurological deficits such as ataxia, paralysis, and stiffness. Signs typically begin caudally and progress cranially. Disease progression is variable and can last days to years.

Treatment includes deworming, anti-inflammatories, and supportive care. Prognosis depends on the severity and duration of disease.

49
Q

Fecal flotation does not work for detection of most trematode eggs because of which one of the following characteristics?

A

Answer: Trematode eggs are too heavy to float.

The fecal flotation technique relies on eggs being less dense than the float solution used.

Most trematode eggs are too dense to float so sedimentation procedures are used to identify trematode eggs.

Trematodes are flukes, such as the lung flukes of small animals, Paragonimus westermani and P. kellicotti.

Link: Parasitology.
https://www.merckvetmanual.com/clinical-pathology-and-procedures/diagnostic-procedures-for-the-private-practice-laboratory/parasitology

50
Q

What correctly describes the classic clinical syndrome associated with equine viral arteritis (EVA)?

A

Answer: Respiratory disease, limb edema, abortion

Equine viral arteritis (EVA) is a contagious viral disease that causes fever, limb edema, abortions, conjunctivitis, and respiratory disease.

Stallions can be silent carriers of the disease and transmit it during breeding. There is a vaccine available that can be used during outbreaks except in pregnant mares and young foals.

Equine Viral Arteritis
https://www.merckvetmanual.com/generalized-conditions/equine-viral-arteritis/equine-viral-arteritis?mredirectid=842

Comprehensive Information on Equine Viral Arteritis for BCSE Test
1. Definitions:
• Equine Viral Arteritis (EVA): A contagious viral disease of equids caused by Equine Arteritis Virus (EAV).
2. Causative Agents:
• Equine Arteritis Virus (EAV): A small, enveloped, positive-sense, single-stranded RNA virus of the family Arteriviridae.

  1. Pathophysiology:
    • Transmission: Direct contact with respiratory secretions, venereal transmission, and indirectly via fomites.
    • Infection Mechanism: EAV infects macrophages and endothelial cells, causing widespread vasculitis. It can also cause abortion by infecting the placenta.
  2. Clinical Signs:
    • Respiratory: Fever, nasal discharge, conjunctivitis, and limb edema.
    • Reproductive: Abortions, usually in the late stages of pregnancy.
    • Neurologic: Ataxia and encephalitis in severe cases.
    • Other: Swelling of the limbs and prepuce, subfertility in stallions.
  3. Diagnosis:
    • Laboratory Tests: Virus isolation, RT-PCR, serology, and immunohistochemistry.
    • Sample Collection: Nasopharyngeal swabs, blood, and tissues from aborted fetuses.
  4. Treatment:• Supportive Care: Antipyretics, anti-inflammatory drugs, and diuretics.
    • Symptomatic Treatment: Reduces fever and swelling.
  5. Prevention and Control:
    • Vaccination: Modified-live vaccines in North America and inactivated vaccines in Europe.
    • Biosecurity Measures: Isolation of infected animals, disinfection of equipment, and management of breeding programs.
  6. Medications:
    • NSAIDs: Used for reducing fever and inflammation.
    • Examples: Flunixin meglumine.
    • Diuretics: To manage edema.
    • Examples: Furosemide.
    Summary of Relevant Information:
    • Causative Agents: Equine Arteritis Virus (EAV).
    • Symptoms: Fever, nasal discharge, conjunctivitis, limb edema, abortions.
    • Clinical Changes: Vasculitis, endothelial damage, placental infection.
    • Assessment: Virus isolation, RT-PCR, serology.
    • Treatment: Supportive care, antipyretics, diuretics.
    • Medications: NSAIDs, diuretics.
51
Q

Pregnant female veterinarians and vet techs need to be particularly cautious about exposure to which organism due to the demonstrated risk of birth defects?

A

Answer: Toxoplasma gondii.

Toxoplasma gondii is the agent of toxoplasmosis, which causes birth defects in a developing fetus if a mother is infected for the first time in her life while pregnant.

It is a zoonotic disease with part of its life cycle in cats. If you are pregnant and you handle cats or clean litter boxes, your developing baby could be at risk if you become infected.

If you were already exposed and had antibodies to toxo prior to pregnancy, then the baby is generally protected by maternal immunity.

However, unless you get a blood test you will not know, so better to be safe and avoid the risk. Some doctors in the U.S. offer testing.

Link: Toxoplasmosis in Animals
https://www.merckvetmanual.com/generalized-conditions/toxoplasmosis/toxoplasmosis-in-animals?autoredirectid=14229

Comprehensive Summary on Toxoplasmosis in Animals for BCSE Test Preparation

Definition and Etiology:
Toxoplasmosis is a zoonotic infection caused by Toxoplasma gondii, an apicomplexan protozoan. It infects all warm-blooded animals, including humans. Cats are the only definitive hosts, shedding oocysts in their feces.

Transmission
• Ingestion: of sporulated oocysts from contaminated environment or tissue cysts in undercooked meat.
• Vertical Transmission: Transplacental transfer from mother to fetus.

Pathophysiology Stages of Infection
• Tachyzoites: Rapidly multiplying form causing tissue damage.
• Bradyzoites: Slow division form in tissue cysts.
• Sporozoites: Infectious form in mature oocysts.
• Replication: After ingestion, the parasite invades intestinal epithelium, replicates, and disseminates via blood and lymph, causing tissue necrosis.

Clinical Findings
• General Signs: Often asymptomatic in immunocompetent hosts.
• Acute Infection: Fever, diarrhea, cough, dyspnea, icterus, seizures, and death in young or immunocompromised animals.
• Reproductive Issues: Abortion, stillbirth in sheep, goats, and pigs.
• Chronic Infection: Subclinical with bradyzoites in tissue cysts.

Diagnosis
• Serologic Testing: Indirect hemagglutination, ELISA, and other serological assays to detect IgM and IgG antibodies.
• Histology and PCR: Identification of tachyzoites or bradyzoites in tissues; PCR for definitive diagnosis.

Treatment
• Anticoccidial Drugs: Sulfadiazine (15–25 mg/kg PO every 12 hours) and pyrimethamine (0.44 mg/kg PO every 12 hours) for 4 weeks.
• Combination Therapy: Trimethoprim-sulfamethoxazole (15 mg/kg PO every 12 hours) for 4 weeks.
• Clindamycin: Preferred treatment for dogs and cats (10–12.5 mg/kg PO for dogs, 25–50 mg/kg PO for cats every 12 hours for 3-4 weeks).
• Other Options: Diaminodiphenylsulfone, atovaquone, spiramycin, toltrazuril, ponazuril, and diclazuril for acute infections and to reduce oocyst shedding in cats.
Prevention and Zoonotic Risk
• Hygiene: Wash hands thoroughly after handling raw meat or contaminated objects. Clean and disinfect surfaces.
• Food Safety: Cook meat to 67°C (152.6°F), avoid raw or undercooked meat, and ensure good kitchen hygiene.
• Pet Care: Feed cats commercially prepared food, clean litter boxes daily, and avoid exposure of pregnant women to cat feces.

52
Q

What percentage of adult cattle with bovine leukosis virus develop lymphosarcoma?

A

Answer: Less than 5%

Many cows have titers to bovine leukosis virus (BLV) but less than 5% develop lymphosarcoma.

Lymphosarcoma is most common in cows 2â6 years old. Tumors can occur in the lymph nodes and tissue almost anywhere in the body.

Clinical signs are dependent on tumor location.

There is no treatment. Risk is minimized by preventing the transfer of blood (and therefore infected lymphocytes) between cows.

Link: Bovine Leukosis
https://www.merckvetmanual.com/generalized-conditions/bovine-leukosis/bovine-leukosis?autoredirectid=20435

Bovine Leukosis: Comprehensive Information for BCSE Test
Definition
• Bovine Leukosis (Bovine Lymphosarcoma): A cancerous disease caused by Bovine Leukemia Virus (BLV), an oncogenic retrovirus.

Etiology and Transmission
• Causative Agent: Bovine Leukemia Virus (BLV), a C-type oncogenic retrovirus.
• Transmission: Primarily horizontal through blood transfer (e.g., contaminated needles, dehorning tools), and rarely vertical (transplacental or via colostrum).

Epidemiology
• Prevalence: High in US dairy herds (~46.5%) and lower in beef herds (~10.3%).
Pathogenesis
• Infection Outcomes:
• Asymptomatic: Most animals show no signs.
• Persistent Lymphocytosis: ~29% develop this benign condition.
• Lymphosarcoma: <5% develop cancerous tumors.
Clinical Findings
• Juvenile Lymphosarcoma: Weight loss, fever, dyspnea, bloat, and posterior paresis.
• Thymic Lymphosarcoma: Cervical swelling, dyspnea, bloat, jugular distention, tachycardia.
• Cutaneous Lymphosarcoma: Cutaneous plaques, enlarged lymph nodes.
Diagnosis
• Serology: ELISA and PCR for BLV detection.
• Cytology/Histology: Biopsy for diagnosing lymphosarcoma.
Treatment and Control
• No Treatment: No effective treatment; symptomatic care with corticosteroids.
• Control Measures: Testing and culling infected animals, disinfection of equipment, using single-use needles.

Prevention
• Blood Precautions: Avoiding exposure to blood from infected animals.
• Colostrum Management: Using colostrum from seronegative cows.
• Farm Practices: Cleaning equipment and facilities, controlling flies.
Zoonotic Risk
• Human Risk: Potential, but not well-established, zoonotic risk through ingestion of unpasteurized dairy products.

53
Q

An adult ewe is presented after she was found wandering aimlessly and pressing her head into the walls of the barn. Now she is recumbent.On presentation the ewe has no menace response. Her pupillary light reflex (PLR) is intact bilaterally and she displays opisthotonos and nystagmus. Which one of the following conditions is the top differential diagnosis?

A

Answer: Poliencephalomalacia

This is most consistent with polioencephalomalacia, which can affect all ruminants sporadically or occasionally in herd outbreaks.

Remember cortical blindness (i.e., no menace, positive PLR bilaterally) in a ruminant is highly suggestive of polioencephalomalacia.

The signs seen with polioencephalomalacia are due to a disruption in cerebral energy metabolism, which causes swelling and necrosis of neurons within the central nervous system.

Can be due to thiamine deficiency, sodium/lead/sulfur toxicosis, or water deprivation. Dx usually based on clinical signs and positive response to thiamine administration.

Listeriosis may cause similar signs of disorientation/recumbency, but usually also unilateral cranial nerve deficits (e.g., head tilt, ear droop) and continuous salivation and food impacted in the cheek of the affected side.
Thrombotic meningoencephalitis (TME) caused by Histophilus somni (formerly Haemophilus somnus) is a common cattle disease in North America which may present with only depression, or sudden death in multiple animals. Histophilosis it is NOT common in sheep though it can occur. Click here to see a report on Histophilus somni in bighorn sheep.

Polioencephalomalacia in Ruminants - Comprehensive Study Guide
Definition
• Polioencephalomalacia (PEM): A neurologic disease in ruminants, characterized by cerebrocortical necrosis.

Causative Agents
• Primary Causes: Thiamine deficiency and high sulfur intake.
• Other Causes: Acute lead poisoning, sodium toxicosis, water deprivation, and toxic plants containing thiaminase.

Pathophysiology
• Thiamine Deficiency: Thiamine is essential for glucose metabolism in the brain. Deficiency leads to impaired glucose metabolism, energy depletion, and neuronal necrosis.
• High Sulfur Intake: Sulfur is reduced to hydrogen sulfide (H2S) in the rumen, which interferes with cellular energy metabolism, particularly in the CNS.

Clinical Signs
• Acute Form: Blindness, recumbency, seizures, coma, high mortality.
• Subacute Form: Separation from group, incoordination, cortical blindness, head pressing, opisthotonos, recumbency, seizures, potentially recovery with minor neurologic impairment.

Lesions
• Gross Lesions: Brain swelling, gyral flattening, cerebellar coning, yellowish discoloration of cortical tissue, autofluorescent necrotic bands.
• Microscopic Lesions: Necrosis of cerebrocortical neurons, cortical spongiosis, vascular hypertrophy and hyperplasia, cavitation of cortical tissue.

Diagnosis
• Clinical Signs: Blindness, head pressing, seizures.
• Necropsy: Autofluorescent bands in the cerebral cortex under UV light.
• Histology: Characteristic neuronal necrosis and cavitation.
• Differential Diagnoses: Lead poisoning, sodium toxicosis, histophilosis, rabies, vitamin A deficiency, pregnancy toxemia, clostridial enterotoxemia, listeriosis.

Treatment
• Thiamine Administration: 10 mg/kg IV initially, then IM for 3-5 days.
• Supportive Therapy: Dexamethasone or mannitol for cerebral edema, symptomatic therapy for seizures.
• Response to Treatment: Improvement usually seen within 24 hours; continued treatment for ≥3 days if no initial response.

Prevention
• Dietary Management: Thiamine supplementation (3-10 mg/kg feed).
• Sulfur Management: Monitor and control sulfur intake from water and feed, especially during hot weather.

54
Q

A castrated male goat is presented for dysphagia and bloat that progressed to generalized stiffness, prolapsed 3rd eyelid, and a “sardonic grin.” When startled, he falls to the ground in an opisthotonos position. The goat was castrated a few weeks ago. Which one of the following choices is the preferred antibiotic for treatment the top differential?

A

Answer: Penicillin is the antibiotic of choice for the treatment of tetanus in ruminants.

Additional treatments include tetanus antitoxin, sedation and/or muscle relaxation, and supportive care in a dark, minimally stimulating environment.

Penicillin used to be the Tx of choice for tetanus in all species.

However, it may have anti-GABA and proconvulsant activities (not ideal in an animal with tetanus!). So, when available, metronidazole is now the Tx of choice HOWEVER it is not legal to use in food animals in North America.

Tetanus may occur sporadically within a herd or there can be larger outbreaks following surgical procedures (ear tagging, castration, tail docking (sheep)) or use of contaminated vaccines or injectable dewormers, all of which create an anaerobic environment in the tissues thay may allow spores of Clostridium tetani to release toxin.

The neurotoxin (tetanospasmin) blocks the inhibitory function of the interneurons of the spinal cord. Lack of inhibitory muscle tone leads to a stiff gait which progresses to stiffness of the head, neck, extremities and tail. Although animals may recover, death from respiratory paralysis is common.

Link: Tetanus in Animals
https://www.merckvetmanual.com/generalized-conditions/clostridial-diseases/tetanus-in-animals

http://www.farad.org/prohibited-and-restricted-drugs.html

Tetanus in Animals - Comprehensive Information for BCSE Test
Definitions and Causative Agent:
• Tetanus (Lockjaw): A neurological disease caused by the neurotoxin produced by Clostridium tetani, an anaerobic bacterium found in soil and intestines.
• Transmission: Introduced through deep puncture wounds, docking, castration, and sometimes through minor or healed wounds.

Pathogenesis:
• Neurotoxin Mechanism: Inhibits inhibitory neurotransmitters by cleaving synaptobrevin, causing muscle spasms, rigidity, and respiratory failure.
• Nerve Pathways: Toxin travels retrograde along motor nerves to the CNS, causing ascending and descending tetanus.

Clinical Findings:
• Symptoms: Localized stiffness, generalized stiffness, tonic spasms, hyperesthesia, difficulty in prehension and mastication (lockjaw), respiratory failure, cardiac arrhythmias, opisthotonos, “sawhorse” stance in horses.
• Incubation Period: Typically 10-14 days.
• Species Differences: Horses, lambs, and humans are highly susceptible; dogs and cats are relatively resistant.

Diagnosis:
• Methods: Clinical signs, recent trauma history, detection of toxin in serum, PCR assay on wound material, gram-stained smears, and anaerobic culture.

Treatment and Control:
• Medications:
• Tetanus Antitoxin: 300,000 IU IV every 12 hours for horses, caution in dogs and cats due to anaphylaxis risk.
• Antibiotics: Penicillin or broad-spectrum antimicrobials.
• Muscle Relaxants: Curariform agents, tranquilizers, barbiturates, diazepam.
• Supportive Care: Wound cleaning, darkened environment, high feeding devices, slings for support.
• Vaccination: Tetanus toxoid for active immunization; booster shots for previously immunized animals; vaccination of mares and foals in high-risk areas.

55
Q

Lymnaeid snails are intermediate hosts for which parasite of ruminants?

A

Answer: Fasciola hepatica

Lymnaeid snails are the intermediate hosts for trematode flukes like Fasciola hepatica and Fascioloides magna.

Link: Fasciola hepatica in Ruminants
https://www.merckvetmanual.com/digestive-system/fluke-infections-in-ruminants/fasciola-hepatica-in-ruminants

https://www.merckvetmanual.com/multimedia/image/fasciola-hepatica-adult

https://en.m.wikipedia.org/wiki/Fascioloides_magna

Fasciola hepatica in Ruminants - Comprehensive Study Guide
Definition
• Fascioliasis: Liver fluke disease caused by Fasciola hepatica, affecting cattle, sheep, alpacas, and llamas.

Causative Agent
• Fasciola hepatica: A liver fluke causing chronic, subacute, and acute infections, with adults residing in bile ducts.

Pathophysiology
• Transmission: Ingestion of metacercariae on vegetation.
• Life Cycle: Eggs hatch in water, miracidia infect snails, asexual development to cercariae, which encyst on vegetation as metacercariae. After ingestion, young flukes excyst in the duodenum, migrate to the liver, and mature in bile ducts.
• Damage: Immature flukes cause hepatic necrosis and hemorrhage, mature flukes cause bile duct fibrosis and calcification.

Clinical Signs
• Acute Disease: Painful abdomen, anemia, sudden death in sheep.
• Subacute Disease: Anemia, hemorrhage, extended survival with hepatic damage.
• Chronic Disease: Anemia, weight loss, decreased milk production, submandibular edema, especially in cattle.

Lesions
• Acute: Enlarged, friable liver with fibrinous deposits.
• Chronic: Cirrhosis, enlarged bile ducts, fibrosis, calcification in cattle.
Diagnosis
• Fecal Sedimentation: Detects eggs.
• ELISA: Detects antibodies in serum or milk.
• Necropsy: Identifies adult flukes and liver damage.
Treatment
• Anthelmintics: Triclabendazole, clorsulon, albendazole, nitroxinil, closantel, rafoxanide, oxyclozanide.
• Resistance: Documented resistance to albendazole, clorsulon, and triclabendazole.
Control
• Pasture Management: Reduce snail habitats, avoid grazing high-risk pastures.
• Snail Control: Molluscicides (historically used, now mostly banned).

56
Q

What diseases is caused by a lentivirus (a slow retrovirus)?

A

Answer: Maedi-Visna is caused by lentivirus.

Maedi-Visna is caused by a lentivirus. It is a slowly progressive disease, usually seen in animals over four years of age. It typically causes pneumonia with respiratory distress. Animals are afebrile and present with weight loss/wasting and ataxia, which progresses to recumbency.

Other lentiviruses of veterinary significance include ovine progressive pneumonia, caprine arthritis and encephalitis, and equine infectious anemia, and bovine and feline immunodeficiency viruses. Lentiviruses are RNA viruses (in the family Retrovirus) known for long incubation periods and chronic conditions that do not have a “cure.”

Louping ill is caused by a flavivirus via tick transmission.

Listeriosis is a bacterial encephalitis.

Link: Lentivirus Pneumonia in Sheep and Goats
https://www.merckvetmanual.com/respiratory-system/respiratory-diseases-of-sheep-and-goats/lentivirus-pneumonia-in-sheep-and-goats?autoredirectid=19855

Link: Caprine Arthritis and Encephalitis
https://www.merckvetmanual.com/generalized-conditions/caprine-arthritis-and-encephalitis/caprine-arthritis-and-encephalitis

Link: Equine Infectious Anemia
https://www.merckvetmanual.com/generalized-conditions/equine-infectious-anemia/equine-infectious-anemia

Link: Louping Ill in Animals
https://www.merckvetmanual.com/nervous-system/louping-ill/louping-ill-in-animals?redirectid=29133

Lentivirus Pneumonia in Sheep and Goats - Comprehensive Study Guide
Definition
• Lentivirus Pneumonia: Also known as Ovine Progressive Pneumonia (OPP) or Maedi-Visna, it is a chronic disease in sheep caused by lentiviruses.

Causative Agents
• Small Ruminant Lentiviruses (SRLVs): Maedi-Visna virus (sheep), Caprine Arthritis Encephalitis virus (goats).

Pathophysiology
• Transmission: Oral ingestion of colostrum/milk or inhalation of infected aerosol droplets. Persistence in lymphocytes, monocytes, and macrophages.
• Infection Mechanism: Virus persists in immune cells, evading elimination and causing a progressive immune response leading to lung and CNS damage.

Clinical Signs
• Sheep: Weight loss, respiratory distress, coughing, bronchial exudate, lethargy, fever, non-inflammatory mastitis (hard bag), encephalitis, neurologic signs (head tilt, circling, altered mentation, paresis).
• Goats: Similar to sheep, including hard bag mastitis and neurologic signs.
Lesions
• Lungs: Mottled, firm, heavy, with rib indentations and enlarged lymph nodes. Histologic interstitial pneumonia, lymphoid hyperplasia, smooth muscle hypertrophy.
• CNS: Meningoencephalitis with demyelination.
Diagnosis
• Serologic Tests: ELISA, AGID assay.
• Advanced Diagnostics: PCR assay, virus isolation, ultrasonography.
Control and Prevention
• Testing and Culling: Regular serologic screening and removal of infected animals.
• Management: Raising lambs/kids from seropositive dams separately, using colostrum/milk from seronegative animals.

57
Q

A three-month-old male shelter puppy is presented with a three-week history of diarrhea with pale, malodorous stools that contain mucus. On physical exam the puppy appears thin but bright and alert. Wet-mounts, shown below, after zinc sulfate fecal flotation reveal cyst-like structures, roughly 12-18 micrometers in length, stained with trichrome. What is the diagnosis?

A

Answer: Giardia lamblia

This is a cyst of the flagellate protozoan Giardia lamblia found in the intestinal tracts of humans and most animals. Most infections do not cause clinical signs, but diarrhea and steatorrhea can occur, particularly in puppies and kittens.

Giardiasis must be differentiated from exocrine pancreatic insufficiency (EPI) and other malabsorption syndromes.

Amebiasis due to Entamoeba histolytica presents as an acute or chronic colitis, with persistent diarrhea or dysentery (mucus, blood in the stool, fever, abdominal pain).

More prevalent in tropical and subtropical countries than in North America. Click here to see an image of Entamoeba histolytica.

Link: Exocrine Pancreatic Insufficiency in Dogs and Cats
https://www.merckvetmanual.com/digestive-system/the-exocrine-pancreas/exocrine-pancreatic-insufficiency-in-dogs-and-cats?redirectid=31744

Link: Malabsorption Syndromes in Small Animals
https://www.merckvetmanual.com/digestive-system/diseases-of-the-stomach-and-intestines-in-small-animals/malabsorption-syndromes-in-small-animals

Link: Entamoeba histolyticaAmebiasis
https://www.merckvetmanual.com/digestive-system/amebiasis/entamoeba-histolytica-amebiasis?autoredirectid=19848

https://www.merckvetmanual.com/multimedia/image/entamoeba-histolytica-cyst

https://www.cdc.gov/dpdx/giardiasis/index.html

58
Q

A one-year-old Angus cow, shown in the image below, is presented with multifocal, circular, alopecic, and crusty lesions, mainly affecting the head and neck. Three other cows have also recently developed similar skin lesions. Dermatophytosis is the suspected problem. Which one of the following choices is the primary dermatophyte in cattle?

A

Answer: Trichophyton verrucosum

Trichophyton verrucosum is the primary dermatophyte in cattle, most commonly seen in calves as periocular lesions although it can become generalized. More common among younger animals and is non-pruritic.

Tx is not usually required as cattle often spontaneously recover.

There are no approved antifungal agents in cattle. Antifungals approved in other animals or humans could be used if a meat and milk withdrawal time can be determined.

Microsporum canis is commonly found in dogs and cats, and Trichophyton mentagrophytes in rodents.

Link: Dermatophytosis in Cattle
https://www.merckvetmanual.com/integumentary-system/dermatophytosis/dermatophytosis-in-cattle

Dermatophytosis in Cattle - Comprehensive Veterinary Information
Definitions and Terminology:
• Dermatophytosis (Ringworm): A fungal infection affecting the skin of cattle.
Causative Agents:
• Primary: Trichophyton verrucosum.
• Others: Trichophyton mentagrophytes, Trichophyton equinum, Microsporum gypseum, Microsporum nanum, Microsporum canis.

Physiopathology:
• Transmission: Direct contact with infected animals or fomites.
• Pathogenesis: Fungi invade keratinized tissues, causing characteristic lesions.
Clinical Findings:
• Lesions: Discrete, scaling patches of hair loss with gray-white crusts.
• Locations:
• Calves: Periocular areas.
• Cows/Heifers: Chest and limbs.
• Bulls: Dewlap and intermaxillary skin.
Diagnosis:
• Clinical Signs: Visual inspection of characteristic lesions.
• Laboratory Tests: Fungal culture, microscopic examination of hair/skin scrapings.
Treatment:
• Topical Therapy: Lime sulfur (1:16), enilconazole (1:100).
• Improving Husbandry: Reducing overcrowding, improving hygiene.
• Vaccination: Live-attenuated fungal vaccine (not available in North America).
Prevention:
• Hygiene: Regular cleaning, burning infective material.
• Isolation: Separate infected animals to prevent spread.

59
Q

A local stable has an outbreak of respiratory disease among the younger foals (conjunctivitis, nasal discharge, anorexia, submandibular edema). Four months later, three mares from the same farm are presented for late-term abortions of fresh fetuses. The mares appear clinically normal otherwise. What disease is the most concerning?

A

Answer: Equine Rhinopneumonitis.

If you hear “nasal discharge” in younger horses and then abortion storms four to five months later in late pregnant mares, think equine herpesvirus-1 (EHV-1, equine rhinopneumonitis). It is the most common infectious cause of abortion in mares.
Vaccinate pregnant mares at five, seven, and nine months gestation to greatly decrease the risk of abortion.

With EVH-1, abortion of a fresh (minimally autolyzed) fetus typically occurs without premonitory signs in the mare. The aborted fetus may have increased thoracic/abdominal fluid, subcutaneous edema, hepatomegaly with numerous white-yellow small foci, and icterus.

Link: Abortion in Horses.
https://www.merckvetmanual.com/reproductive-system/abortion-in-large-animals/abortion-in-horses

Abortion in Horses - Comprehensive Information for BCSE Test
Definitions and Etiology:
• Abortion: Premature expulsion of the fetus, which can be caused by various infectious and noninfectious agents.

Noninfectious Causes:
1. Twinning: Common cause, typically results in abortion at 8-9 months due to placental insufficiency.
2. Umbilical Cord Torsion: Common in Thoroughbreds, diagnosed by localized swelling or hemorrhage.
3. Fescue Grass Toxicosis: Caused by ingestion of Epichloë coenophiala; leads to prolonged gestation, agalactia, and placental separation.

Mare Reproductive Loss Syndrome (MRLS): Associated with exposure to eastern tent caterpillars, causes abortions, stillbirths, and weak foals. Pathology includes neutrophilic placentitis and funisitis. Symptoms: Early abortions (40-80 days), late-term abortions (10 months-term), stillbirths, and weak foals. Causes: Associated with ingestion of eastern tent caterpillars, leading to bacteremia and fetal loss.

Infectious Causes:
1. Bacterial Abortion:
• Common Agents: Streptococcus equi, Escherichia coli, Pseudomonas aeruginosa, Leptospira spp.
• Symptoms: Premature udder development, mucopurulent vaginal discharge.
• Diagnosis: Isolation from placenta, fetal tissues.
• Treatment: Antibiotics, hygiene.
2. Equine Mycotic Placentitis:
• Agents: Aspergillus, Mucor, Candida.
• Symptoms: Late-term abortion, thickened chorioallantois.
• Diagnosis: Presence of hyphae in placenta, fetal tissues.
• Treatment: Antifungal therapy.
3. Equine Rhinopneumonitis (EHV-1):
• Symptoms: Abortion after 7 months, edematous placenta, jaundice in fetus.
• Diagnosis: PCR, virus isolation from fetal tissues.
• Prevention: Vaccination at 5, 7, and 9 months gestation.
4. Equine Viral Arteritis (EVA):
• Symptoms: Abortion 6-29 days post-infection, fetal arteritis.
• Diagnosis: PCR, virus isolation from placenta, fetal tissues.
• Prevention: Vaccination, management to prevent viral transmission.
Diagnosis and Prevention
• Diagnosis: Clinical signs, laboratory tests (culture, PCR, serology).
• Prevention: Management practices, vaccination, vector control, and avoiding exposure to known infectious agents.

60
Q

A nursing lamb has multiple vesicles, pustules, and large coalescing crusts on the lips and oral mucosa. The ewe has large friable crusts and granulation tissue on the teats and udder. What is the most likely diagnosis?

A

Answer: Contagious Ecthyma.

If you hear “crusty lips/ears” on a goat or sheep, think of contagious ecthyma, also called “orf” or “soremouth.”
Caused by a parapoxvirus which enters the body through cuts. See papules, pustules, and thick friable scabs on the lips, mouth, nose, ears, and feet; and on the udders of lactating does and ewes.

Treatment is supportive. Orf/soremouth is common, very contagious, and zoonotic, so be careful when you handle infected animals, as humans are also at risk of developing crusty sores. In endemic areas, young animals are most susceptible.

Goat and sheep-pox are serious, foreign animal diseases. Follow this link to see sheep-pox: NOT found in North America and reportable.

For good refs on Foreign Animal Diseases (FADs), download the newly revised 7th edition of The Gray Book (Note- this is a large PDF file, it will take a minute to load), available free online courtesy of the U.S. Animal Health Association.

For a fast online search, try the Center for Food Security and Public Health (CSFPH) Zoonosis website.

Link: Contagious Ecthyma in Sheeps and Goats.
https://www.merckvetmanual.com/integumentary-system/pox-diseases/contagious-ecthyma-in-sheep-and-goats?autoredirectid=21714

https://www.merckvetmanual.com/multimedia/image/sheeppox-lesions

Link: Foreign Animal Diseases
https://www.usaha.org/upload/Disease Info/FAD.pdf

Link: Zoonotic Diseases
https://www.cfsph.iastate.edu/zoonoses/

Contagious Ecthyma in Sheep and Goats - Detailed Veterinary Summary
Etiology and Pathogenesis
• Causative Agent: Parapoxvirus
• Transmission: Direct contact, fomites; virus highly resistant in the environment
• Pathogenesis: Entry through skin abrasions, viral replication in keratinocytes causing cytoplasmic swelling and degeneration, leading to vesicle formation and subsequent pustules.

Clinical Findings
• Symptoms: Papules, vesicles, pustules primarily on lips, face, feet, teats, occasionally buccal cavity and feet; anorexia, weight loss, secondary infections
• Lesions: Progress from macules to papules, vesicles, pustules, and scabs; most common at mucocutaneous junctions

Diagnosis
• Tests:
• PCR: Gold standard for detecting viral DNA in scabs
• Electron Microscopy: Identification of characteristic viral particles
• Histopathology: Intracytoplasmic inclusion bodies
• Differential Diagnosis: Foot-and-mouth disease, bluetongue, ulcerative dermatosis
Treatment and Control
• Medications:
• Antimicrobials: Prevent secondary bacterial infections
• Topical Treatments: Prevent myiasis, soothing ointments for lesions
• Vaccination: Live virus vaccines, typically prepared from infected scab material; administered by scarification, providing immunity but with caution due to zoonotic risk

Prevention
• Hygiene and Management: Regular disinfection of equipment, isolation of infected animals, proper handling of infected materials
• Biosecurity: Prevent introduction from new animals, quarantine measures
• Environmental Control: Reducing contamination of grazing areas, maintaining dry conditions to minimize virus persistence

Key Points
• Zoonotic Potential: Can cause orf in humans, characterized by painful pustules, particularly on hands; use of gloves and proper hygiene essential during handling
• Resilience: Virus can survive in scabs for years; thorough cleaning and disinfection required to prevent environmental persistence
• Public Health Considerations: Inform handlers of zoonotic risk, ensure proper wound care and protection when dealing with infected animals

61
Q

A budgerigar is presented that looks like the image below. What is the diagnosis?

A

Answer: Knmidocoptes pilae.

This is Knemidocoptes pilae, the scaly face (or leg) mite and this appearance is considered pathognomic. Confirm Dx with facial scraping if indicated.

Tx with ivermectin (0.2 mg/kg IM or PO) or moxidectin (0.2 mg/kg PO) repeated in two weeks. Tx is generally curative but beak deformity may persist.

In passerine birds (esp. canary, European goldfinch), look more for crusts on legs, digits (“tassel foot”). Do not perform leg scrapings for diagnosis as this can result in excessive hemorrhage.

Link: Parasitic diseases of Pet Birds.
https://www.merckvetmanual.com/exotic-and-laboratory-animals/pet-birds/parasitic-diseases-of-pet-birds

62
Q

A small kitten is presented with an open wound in the right ventral neck containing a 1.2-inch (3-cm) long parasite covered in tiny black spines. What is the organism?

A

Answer: Cuterebra.

Cuterebra (a.k.a. rabbit botfly larva) is occasionally found in outdoor-roaming cats and dogs. In North America, cases are usually seen during late summer and fall. The visible fistula is a breathing hole for the larva which eventually exits, pupates and becomes a fly.

The normal hosts are rabbits, squirrels, chipmunks and mice. Cuterebra flies lay eggs near the nests, burrows, and traffic pathways where they can attach to the body of hosts. The eggs hatch in response to body heat and larvae enter the body via the mouth or nostrils during grooming, but may also enter through a wound. They penetrate the tissues and travel to specific subcutaneous areas where they mature.

Extract the larva carefully in one piece to avoid a severe local inflammatory reaction that can result from a ruptured grub. Do not squeeze the area as that might damage the larva.

Anaphylaxis in response to rough removal has also been reported anecdotally.

If there is only a swelling without an obvious breathing hole (yet), an abscess or foreign body are ruleouts for this lesion.

Cattle grubs (Hypoderma spp.) are generally lighter colored with less prominent spines and are rarely found in pets.

The larvae that cause cutaneous habronemiasis are tiny - they emerge from infected flies feeding on wounds and invade the damaged tissue.

Horse bots (Gasterophilus spp.) live in the stomach and are not known to infect dogs and cats.
ies feeding on wounds and invade the damaged tissue.

Sheep nose bots (Oestrus ovis) are smaller than cuterebrae larva. They live in the nares and only affect sheep and goats.

  1. Cuterebra larvae: seen in outdoor cats/dogs, late summer/fall.
  2. Larvae enter through grooming wounds, mature subcutaneously.
  3. Extract larvae carefully to avoid severe inflammation, anaphylaxis.

Link: Cuterebra infestation in Dogs and Cats.
https://www.merckvetmanual.com/integumentary-system/cuterebra-infestation-in-dogs-and-cats/cuterebra-infestation-in-dogs-and-cats

Link: Hypoderma spp.
https://www.merckvetmanual.com/integumentary-system/cattle-grubs/hypoderma-spp

Link: Cutaneous Habronemiasis in Animals.
https://www.merckvetmanual.com/integumentary-system/helminths-of-the-skin/cutaneous-habronemiasis-in-animals

Link: Gasterophilus spp Infection in Horses
https://www.merckvetmanual.com/digestive-system/gastrointestinal-parasites-of-horses/gasterophilus-spp-infection-in-horses?redirectid=27858

Link: Sheep Nasal bot myiasis
https://www.merckvetmanual.com/respiratory-system/respiratory-diseases-of-sheep-and-goats/sheep-nasal-bot-myiasis?autoredirectid=19854

63
Q

A 12-pound 10-day-old female goat kid is presented for watery diarrhea and weakness.
Temperature is 100°F (37.5°C) [N=102-104°F (39.5-40.5°C)]
Heart rate is 180 bpm [N=90-150 bpm]
Respiratory rate is 50 brpm [N=20-40 brpm]
Skin tenting is 3 seconds and there is moderate enophthalmos.
The kid is approximately 8% dehydrated based on physical exam findings. You develop a fluid therapy plan but first need to estimate the fluid deficit from dehydration.
What is this goat kid’s estimated fluid deficit?

A

Answer: 0.4 L

This goat kid needs 0.4 liters of fluid (~400 ml) to replace its dehydration deficit. The calculation is as follows:
Weight in kgs x % dehydration = fluid deficit in liters; 8% dehydration=0.08
12lbs à 2.2 kg/lb = 5.45 kg
5.45 kg x 0.08 dehydration = 0.43 liters

Replacement fluids should include electrolytes and bicarbonate in kids with diarrhea. Some glucose and potassium support may also be necessary.

Fluids can be administered IV with an isotonic solution over 3-4 hours. Kids are notorious for chewing IV lines, so either monitor closely or use an Elizabethan collar.

An alternative to isotonic IV fluids for diarrhea is IV hypertonic saline or IV hypertonic sodium bicarbonate with oral electrolytes.

This only takes a few minutes and is as just as effective.

Link: The Fluid Resuscitation Plan in Animals
https://www.merckvetmanual.com/emergency-medicine-and-critical-care/fluid-therapy/the-fluid-resuscitation-plan-in-animals

The Fluid Resuscitation Plan in Animals

Concepts
Fluid resuscitation aims to restore intravascular volume and improve tissue perfusion in animals experiencing shock or severe dehydration.

Etiology
• Hypovolemic Shock: Often due to hemorrhage, severe dehydration, or third-space fluid losses.

Pathogenesis
• Shock Response: Compensatory mechanisms, including increased heart rate and vasoconstriction, aim to maintain perfusion but may become exhausted, leading to decompensatory shock and cellular hypoxia.

Symptoms
• Hypovolemic Shock: Tachycardia, weak pulse, prolonged capillary refill time, pale mucous membranes, altered consciousness.

Diagnostic Methods
• Clinical Assessment: Heart rate, pulse intensity, mucous membrane color, capillary refill time, blood pressure, rectal temperature.
• Laboratory Tests: PCV, total solids, blood lactate, electrolytes, and osmolality.
Treatment
• Fluid Selection:
• Crystalloids: Isotonic solutions (e.g., lactated Ringer’s, 0.9% saline) for interstitial volume replacement.
• Colloids: Synthetic or natural colloids (e.g., hydroxyethyl starch, albumin) for intravascular volume expansion.
• Fluid Administration Techniques:
• Large-Volume Resuscitation: Rapid infusion of isotonic crystalloids followed by colloids for hypovolemic shock.
• Small-Volume Resuscitation: Used in cases where large volumes could cause complications (e.g., brain or lung edema).

Determination of Resuscitation Endpoints
• Clinical Parameters: Restoration of normal heart rate, blood pressure, capillary refill time, and mucous membrane color.
• Supplementary Endpoints: Central venous pressure, central venous oxygen saturation, and urine output.

Special Considerations
• Electrolyte Management: Careful monitoring and adjustment of sodium and potassium levels to avoid complications like cerebral edema or cardiac arrhythmias.
• Individualized Approach: Consideration of renal function, ongoing losses, and presence of concurrent diseases (e.g., heart disease, brain injury).

64
Q

A silicone gastrostomy tube can typically be used for what period of time?

A

Answer: 8-12 months.

Gastrostomy tubes made of silicone can last 8-12 months. Latex tubes do not last more than about 12 weeks as the latex eventually breaks down from exposure to stomach acids.

A gastrostomy tube is a feeding tube that passes through the abdominal wall into the stomach. These allow placement of food directly into the stomach and are used when the patient is unable or unwilling to eat normally because of damage to the esophagus, head, or neck.

Often called “PEG” (percutaneous endoscopic gastrostomy) tubes because they are placed using an endoscope.

65
Q

How do you measure the quantitative immune response to a virus?

A

Answer: Serum titer

A serum titer (serology.) measures the amount of antibody specific to a particular antigen (or virus) within the blood. It is the most specific way to measure an immune response to a particular virus.

Polymerase chain reaction (PCR) looks for particular DNA; it can determine the presence of a virus but NOT the animal’s immune reaction to that virus.

Changes in the lymphocyte count, neutrophil count, and serum amyloid A all can occur with viral infections but are very nonspecific and are not quantitative.

66
Q

Episodes of hyperkalemic periodic paralysis (HYPP) can be prevented in affected horses by which treatment?

A

Answer: Treatment with acetazolamide

Treat with acetazolamide PO daily and feed Timothy or Bermuda grass diet to help prevent hyperkalemic periodic paralysis (HYPP) episodes. HYPP is an autosomal dominant trait resulting from a point mutation causing a defect in a voltage dependent skeletal muscle sodium channel. This mutation has been linked to the popular Quarter Horse stallion “Impressive.”

Affected horses can suffer intermittent episodes of muscular fasciculations and weakness of variable severity. Onset of signs unpredictable, and many triggers: diets high in POTASSIUM (>1.1%, ie: alfalfa hay, molasses, electrolyte supplements, kelp-based supplements). Fasting, anesthesia, heavy sedation, trailer rides, and stress can precipitate clinical signs.

Treatment and prevention of HYPP based on DECREASING POTASSIUM in blood. Remember: high K+ SLOWS the HEART (even unto death). Usually (but not always) see INCREASED K+> 5 mEq/L (normal 3-5) during an episode, but normal between episodes.

PREVENTION based INCREASING renal excretion of K+ with acetazolamide (carbonic anhydrase inhibitor more assoc. with glaucomaTx) and feeding a low K+ diet (i.e., use Timothy or Bermuda hay, AVOID alfalfa or brome hay).

Link: Congenital and Inherited Anomalies of the Musculoskeletal System in Horses
https://www.merckvetmanual.com/musculoskeletal-system/congenital-and-inherited-anomalies-of-the-musculoskeletal-system/congenital-and-inherited-anomalies-of-the-musculoskeletal-system-in-horses

Link: Hyperkalemia in Ruminants
https://www.merckvetmanual.com/metabolic-disorders/disorders-of-potassium-metabolism/hyperkalemia-in-ruminants?autoredirectid=14269

Congenital and Inherited Anomalies of the Musculoskeletal System in Horses
Angular Limb Deformities
• Etiology: Utero malposition, hypothyroidism, trauma, poor conformation.
• Symptoms: Lateral/medial deviation of limbs, lameness, soft-tissue swelling.
• Diagnosis: Radiography for physeal, epiphyseal, and carpal bones.
• Treatment: Tube casts, splints, surgical correction (hemicircumferential transection, periosteal elevation).

Digit Malformation
• Types: Polydactyly, hypoplasia, bipartite/tripartite navicular bone.
• Treatment: Surgical correction for functional impairment.
Dwarfism
• Types: Proportionate (growth hormone deficiency) and disproportionate (thyroid hormone issues).
• Symptoms: Delayed cuboidal bone development, large head, silky coat.
• Diagnosis: Clinical evaluation, hormonal assays.
Glycogen Branching Enzyme Deficiency
• Etiology: Enzyme deficiency affecting glycogen storage.
• Symptoms: Stillbirth, seizures, cardiac/respiratory failure.
• Diagnosis: Blood tests for GBE activity, muscle/organ biopsies.
• Treatment: No effective treatment, fatal disease.
Hernias
• Types: Umbilical, diaphragmatic, inguinal.
• Symptoms: Scrotal swelling, colic.
• Treatment: Surgical repair for larger or symptomatic hernias.
Hyperkalemic Periodic Paralysis (HYPP)
• Etiology: Genetic mutation in sodium channels.
• Symptoms: Paralysis, collapse, sudden death.
• Diagnosis: Genetic testing.
• Treatment: Manage with diet and medication.
Spine Defects
• Types: Scoliosis, synostosis, lordosis, kyphosis, torticollis.
• Symptoms: Gait abnormalities, back weakness.
• Diagnosis: Clinical examination, radiography.
• Treatment: Often conservative, severe cases may need management.
Tying-up Syndrome
• Etiology: Abnormal calcium regulation, polysaccharide storage myopathy (PSSM).
• Symptoms: Muscle cramping, stiffness.
• Diagnosis: Genetic testing, muscle biopsy.
• Treatment: Dietary management, regular exercise.

67
Q

What effect can thiamine deficiency have on the feline patient?

A

Answer: Cervical ventroflexion.

Cervical ventroflexion can be seen in cats with thiamine deficiency, as well as other neurologic signs like blindness, ataxia, weakness, tremors, seizures and coma. Gl signs are typically seen first. If left untreated, this can be fatal.

Deficiency of thiamine (Vitamin B1) is most often seen in cats eating an unbalanced homemade diet; raw seafood contains thiaminases that break down thiamine.

Thiamine deficiency, hypokalemia, and taurine deficiency are all possible nutritional causes of cervical ventroflexion in cats.

https://www.merckvetmanual.com/nervous-system/diseases-of-the-spinal-column-and-cord/nutritional-disorders-of-the-spinal-column-and-cord-in-animals?autoredirectid=16748#Thiamine-Deficiency_v26305230

Link: Feline Hypokalemic Polymyopathy
https://www.merckvetmanual.com/musculoskeletal-system/myopathies-in-small-animals/feline-hypokalemic-polymyopathy

Thiamine Deficiency in Animals
Thiamine (vitamin B1) deficiency is most common in cats; however, it has also been reported in dogs. Causes include inadequately formulated commercial diets, vegetarian diets, food preserved with sulfur dioxide (which destroys thiamine), and raw fish diets (which contain thiaminase). Affected cats typically exhibit brain dysfunction characterized by vestibular signs, head tremor, ataxia, depression, severe ventroflexion of the head, seizures, and death. Clinical signs in dogs include anorexia, depression, paraparesis, seizures, coma, and death. Pathological findings are polioencephalomalacia, most prominent in the midbrain. Diagnosis is based on clinical signs, dietary history, and response to thiamine administration (thiamine hydrochloride, 10-20 mg/cat or 25-50 mg/dog, IM, every
24 hours for 2-4 weeks or until clinical signs abate).

68
Q

A 4-year-old male neutered domestic shorthair cat is presented for urethral obstruction. Treatment includes placement of an indwelling urinary catheter and fluid diuresis for 48 hours. Upon removal of the urinary catheter, the cat makes frequent attempts to urinate with only minimal urine output. Manual bladder expression is difficult. Urethral spasm is suspected. What is an appropriate treatment for urethral spasm?

A

Answer: Phenoxybenzamine

Treat urethral spasm, a common complication following treatment of urethral obstruction, with phenoxybenzamine or prazosin. Both drugs reduce urethral sympathetic tone.

Use phenylpropanolamine and/or estriol for urethral sphincter mechanism incompetence, a common cause of urinary incontinence in female dogs.

Sulfasalazine reduces large bowel inflammation.

Sotalol is a non-selective beta blocker and Class III antiarrhythmic used to treat ventricular tachycardia.

Link: Obstructive Uropathy in Small Animals
https://www.merckvetmanual.com/urinary-system/noninfectious-diseases-of-the-urinary-system-in-small-animals/obstructive-uropathy-in-small-animals

Link: Pharmacotherapeutics in Urinary Incontinence in Dogs and Cats
https://www.merckvetmanual.com/pharmacology/systemic-pharmacotherapeutics-of-the-urinary-system/pharmacotherapeutics-in-urinary-incontinence-in-dogs-and-cats?autoredirectid=22714#v3333479

Obstructive Uropathy in Small Animals - Comprehensive Study Guide
Definition
• Obstructive Uropathy: Blockage of urine flow at any point below the kidneys, leading to postrenal azotemia and uremia.

Causative Agents
• Common Causes: Uroliths (bladder stones) in dogs, matrix-crystalline plugs in young male cats, ureteral uroliths in geriatric cats, tumors, blood clots.

Pathophysiology
• Mechanism: Obstruction leads to increased hydrostatic pressure in the renal pelvis, causing hydronephrosis, hydroureter, renal parenchyma atrophy, and cystic enlargement.
• Acute Complete Obstruction: Rapid onset of azotemia and uremia due to inability to excrete waste.
• Partial or Unilateral Obstruction: Compensatory hypertrophy of the nonaffected kidney may mask clinical signs.

Clinical Signs
• Urethral Obstruction: Pollakiuria, stranguria, hematuria, abdominal pain, distended painful bladder, inability to pass a catheter.
• Uremia Signs: Vomiting, dehydration, hypothermia, severe depression.
• Severe Hyperkalemia: Bradycardia, cardiac arrhythmias (tall T waves, widened QRS complex).
Diagnosis
• Clinical Signs and History: Physical examination findings, particularly inability to pass a urethral catheter.
• Laboratory Tests: Serum biochemistry (elevated potassium, azotemia), ECG for hyperkalemia.
• Imaging: Excretory urography, abdominal ultrasonography to confirm ureteral obstruction.
Treatment
• Immediate Relief of Obstruction: Urethral catheterization, removal of ureteral obstructions via surgery if necessary.
• IV Fluids: Normal saline to improve renal function and correct electrolyte imbalances.
• Medications for Hyperkalemia:
• Bicarbonate: 0.5 mEq/kg IV over 5 minutes.
• Insulin and Dextrose: To drive potassium intracellularly.
• Monitoring: Daily assessment of plasma electrolytes, body weight, urine output, hematocrit, and plasma total solids.

69
Q

A three-year-old female spayed mixed breed dog is presented for recheck of idiopathic epilepsy diagnosed one month previously. The dog has had two seizures in the past two weeks and is currently receiving phenobarbital at 3 mg/kg PO q 12 h and potassium bromide at 30 mg/kg PO q 24 h. Physical exam is unremarkable. What is the most appropriate next step?

A

Answer: Discontinue phenobarbital in this dog.

Discontinue phenobarbital in this dog with neutropenia, thrombocytopenia, and mild anemia. This is a rare side effect of treatment, seen most commonly in the first three months of treatment. Monitor the CBC during this period because changes are typically reversible when detected early.

Taper phenobarbital over seven days to reduce the risk of breakthrough seizures, which could result in status epilepticus.
Add another anti-epileptic drug (e.g., levetiracetam) in this dog because of continued seizures and the need to discontinue phenobarbital.

Monitor liver enzymes and serum phenobarbital concentration every 6 to 12 months in dogs receiving chronic phenobarbital therapy because liver enzyme increase is common. While it does not necessarily indicate hepatic dysfunction, ALT or ALP > 5 x reference range, increased GGT, hyperbilirubinemia, or increased serum bile acid concentration is concerning for phenobarbital-induced hepatic dysfunction.

If these chemistry changes occur consider tapering/discontinuing phenobarbital and adding another anti-convulsant. Hepatic dysfunction/failure are typically seen with higher serum phenobarbital concentrations.

Other potential side effects of phenobarbital include polyuria/ polydipsia/polyphagia. Phenobarbital can affect the results of thyroid testing and falsely lower total and free T4 and increase TSH. Dogs with these findings do not have hypothyroidism, so do not initiate Tx based on thyroid hormone measurement in patients on phenobarbital.

Link: Phrnobarbital
https://www.merckvetmanual.com/pharmacology/systemic-pharmacotherapeutics-of-the-nervous-system/maintenance-anticonvulsant-or-antiepileptic-therapy#Phenobarbital:_v4502428

Phenobarbital:
Phenobarbital has a long record of safety, efficacy, low cost, and convenience in regard to monitoring serum concentrations. For longterm maintenance in cats and dogs, phenobarbital may be given at 2-4 mg/kg/day, PO, bid.
In all species, it takes ~2 wk to approach a steady-state plasma concentration, because oral absorption is extremely variable and the half-life is long. Drug levels are monitored 2 wk after initiation of therapy, 2 wk after any dosage change, and usually every 6-12 mo once seizure control is achieved. Serum therapeutic concentrations are 15-45 mcg/mL. The dosage should be adjusted on the basis of serum level and the history of seizure control. Tolerance to phenobarbital therapy may develop in dogs treated continually for months to years and may result in decreased seizure control; however, an increase (25%) in the dose usually will result in improved seizure control.
Phenobarbital causes physical dependence, and thus abrupt withdrawal may cause “barbiturate withdrawal” seizures. Hepatotoxicity and liver failure in dogs have been associated with high serum concentrations (>35 mcg/mL), necessitating the serum level checks every 6-12 mo. Adverse effects such as sedation, polydipsia, polyuria, and polyphagia are common but may decrease within the first few weeks. Other, less common adverse effects are idiosyncratic hyperexcitability, dermatitis, anemia, neutropenia, thrombocytopenia, gingival hyperplasia, and osteomalacia. Phenobarbital has also been used to treat episodic dyscontrol syndrome (rage) in dogs when seizure activity is demonstrable via electroencephalogram recordings.
Oral phenobarbital has been used in ruminants at 11 mg/kg/day and in horses at 3-11 mg/kg/day. Serum concentrations should be checked periodically.

70
Q

Most antimicrobials are toxic to small mammals such as rodents (e.g., guinea pigs, hamsters) and rabbits. Which set of antimicrobials are safe for use in these species?

A

Answer: Trimethoprim sulfa, chloramphenicol, enrofloxacin

Use of the antimicrobials enrofloxacin, trimethoprim sulfa, and chloramphenicol is considered safe in rabbits and rodents.
Avoid penicillins (e.g., amoxicillin) in these species and avoid clindamycin in rabbits. Always review the safety profile and dosing recommendations for drugs in small mammals to ensure safety and efficacy.

Rabbits, hamsters, and other rodents are particularly sensitive to enteric dysbiosis +/- enterotoxemia. Antimicrobials may disturb their normal gut flora, particularly the beneficial gram-positive bacteria which digest high-fiber food and keep pathogenic clostridial organisms from over-growing.

Remember: Chloramphenicol causes a potentially fatal aplastic anemia in humans, so always use with caution and provide appropriate client information.

Link: Clinical Pathology in Rabbits
https://www.merckvetmanual.com/exotic-and-laboratory-animals/rabbits/management-of-rabbits#Clinical-Pathology_v3306361

Comprehensive Overview of Rabbit Management for BCSE Test Preparation
Definitions and General Management
• Housing: Adequate space, proper ventilation, and temperature control are critical.
• Diet: High-fiber diet including hay, fresh vegetables, and water.
• Breeding: Understanding reproductive cycles and proper care for does and kits.
Common Diseases and Conditions
• Gastrointestinal Stasis: Symptoms include anorexia, lethargy, and reduced fecal output.
• Diagnosis: Clinical signs, radiographs showing gas-filled stomach and intestines.
• Treatment: Fluid therapy, motility drugs, and dietary management.
• Pasteurellosis: Caused by Pasteurella multocida.
• Symptoms: Respiratory signs, abscesses, and otitis media.
• Diagnostics: Culture and sensitivity tests.
• Treatment: Antibiotics like enrofloxacin or doxycycline.
• Urolithiasis: Formation of bladder stones.
• Symptoms: Hematuria, dysuria, and abdominal pain.
• Diagnostics: Radiographs and urinalysis.
• Treatment: Surgical removal, dietary adjustments to reduce calcium intake.

Neoplasias in Rabbits
• Uterine Adenocarcinoma: Common in older does.
• Symptoms: Hematuria, abdominal masses.
• Diagnostics: Ultrasound and histopathology.
• Treatment: Ovariohysterectomy.
• Lymphoma: Affects multiple organ systems.
• Symptoms: Weight loss, lethargy, lymphadenopathy.
• Diagnostics: Biopsy and immunohistochemistry.
• Treatment: Chemotherapy, though prognosis is poor.

Clinical Pathology and Diagnostics

•	Hematology and Biochemistry: Normal values vary slightly from other small animals.
•	Common Tests: CBC, serum biochemistry, urinalysis.
•	Interpretations: Specific changes like elevated calcium in renal disease, or changes in liver enzymes in hepatic conditions.
•	Imaging: Radiographs, ultrasound for internal organ assessment.
•	Endoscopy: For gastrointestinal and respiratory tract examination.

Summary Points for BCSE Test
• Housing and Diet: Essential for preventing common diseases.
• Disease Management: Identification, diagnosis, and treatment of gastrointestinal stasis, pasteurellosis, and urolithiasis.
• Neoplasias: Recognition and management of uterine adenocarcinoma and lymphoma.
• Clinical Pathology: Understanding normal values and interpretation of common diagnostic tests.

71
Q

What does the acronym TPN mean stand for in veterinary medicine?
Temporal Parietal Nodule
Titrated Pediatric Nephrosis
Triple Phosphate Neomicin
Total Parental Nutrition
Temporary Patient Number

A

Answer: Total Parental Nutrition.

TPN is Total Parenteral Nutrition, which is giving all the nutrition an animal needs through a route other than the Gl tract (ie: administered via IV route).

Animals suffering from severe anorexia may be fed this way.

Refs: Bassert and Thomas, McCurnin’s Clinical Textbook for Veterinary Technicians, gth ed. p. 334.

Link: Nutrition in Disease Management in Small Animals

72
Q

What is the physiologic basis of Von Willebrand’s disease?
Megakaryocyte dysplasia
Autoimmunity to sub endothelial glycosaminoglycans
Coagulation factor deficiency
Factor X, VII defect
Congenital platelet defect

A

Answer: Coagulation factor deficiency.

Von Willebrand’s factor is needed for the first step in clot formation, and animals with von Willebrand’s disease are deficient in this factor. In normal animals, von Willebrand’s factor circulates in a complex with coagulation Factor VIlI and facilitates adhesion of platelet to subendothelial surfaces.

Suspect von Willebrand’s disease in dogs with bleeding disorders with a lab pattern of normal platelet counts and across the board normal coagulation tests. It is the most common inherited coagulation disorder in dogs.

Signs include gingival bleeding, epistaxis, and hematuria. Disease may go undetected in a puppy until an excessive bleeding episode after vaccination injection, venipuncture, or surgery (ie: tail docking, dewclaw removal).

Chediak-Higashi syndrome, is due to a congenital platelet defect. Associated with dilute hair color-think of smoke-blue Persian cats and beige mice. Not seen in dogs.

Pelger-Huet anomaly is a thrombocyte and white cell defect. Granulocytes, especially neutrophils are stuck in band form. A CBC shows an apparent left shift with a normal WBC count. Usually an incidental finding because cell function is normal.

Click here for an image of Pelger-Huet anomaly in a dog.
https://www.merckvetmanual.com/multimedia/v12762869

Ref: Blackwell’s 5-Min. Vet Consult Canine-Feline, 4th ed. pp. 1436-37.

Link: Von Willebrand Disease in Animals
https://www.merckvetmanual.com/circulatory-system/hemostatic-disorders/platelet-disorders-in-animals#v3258487

Link: Chèdiak-Higashi Syndrome in Animals
https://www.merckvetmanual.com/circulatory-system/hemostatic-disorders/platelet-disorders-in-animals#v3258495

Link: Leukogram Abnormalities in Animals

Von Willebrand Disease in Animals
Definitions:
• Von Willebrand Disease (vWD): Inherited bleeding disorder due to defective or deficient von Willebrand factor (vWF).

Causative Agents:
• Deficient or defective vWF.
Physiopathology:
• vWF mediates platelet adhesion.
• vWF deficiency leads to impaired clotting.
Symptoms:
• Mucosal bleeding, gingival bleeding, epistaxis, hematuria, prolonged bleeding after surgery.

Clinical Changes:
• Type 1: Mild to moderate, low vWF concentration.
• Type 2: Moderate to severe, loss of high-molecular-weight multimers.
• Type 3: Severe, total absence of vWF.
Assessment:
• Clinical signs, prolonged buccal mucosal bleeding time, PFA-100 closure times.
• Laboratory tests: Low vWF concentration, prolonged APTT and ACT in some cases.
Treatment:
• Avoid drugs that interfere with platelet function.
• Transfusion of cryoprecipitate, fresh frozen plasma, or whole blood.
• Desmopressin acetate (1 mcg/kg SC 30 minutes before surgery).
Medication:
• Desmopressin Acetate: Increases vWF release.
• Transfusions: Cryoprecipitate, fresh frozen plasma, fresh whole blood.
Inheritance:
• Autosomal recessive and autosomal dominant patterns.
Prevalence:
• High in certain dog breeds.

Chédiak-Higashi Syndrome in Animals
Definitions:
• Chédiak-Higashi Syndrome: Autosomal recessive disorder with abnormal granule formation in leukocytes, melanocytes, and platelets.
Physiopathology:
• Microtubule formation defect; large but reduced granules in cells.
Symptoms:
• Prolonged bleeding, decreased leukocyte function, diluted coat color.
Clinical Changes:
• Abnormal granules in various cells, decreased platelet aggregation and storage of adenosine diphosphate and serotonin.

Assessment:
• Clinical signs, microscopic examination of cells.
Prevalence:
• Diagnosed in mink, cattle, beige mice, and blue smoke Persian cats.

Pelger-Huët Anomaly
Definitions:
• Pelger-Huët Anomaly: Nuclear hyposegmentation defect of granulocytes.
Physiopathology:
• Neutrophils appear as bands and metamyelocytes.
Symptoms:
• None in heterozygotes; lethal in homozygotes.
Clinical Changes:
• Hyposegmented nuclei in granulocytes.
Assessment:
• Blood smear analysis showing nuclear hyposegmentation.
Prevalence:
• Reported in people, cats, rabbits, horses, and dogs.

73
Q

Which choice is a normal capnography reading for a patient under inhalant anesthesia?

0-10 mg Ag
20-30 mmH20
75 - 85 ml/min
40 - 55 mmHg
95-100%

A

Answer 40-55mmHg.

Capnography measures the amount of expired carbon dioxide (COn) in each breath. This is commonly called’end-tidal CO2! The level should rise to 40-55 mm Hg at peak expiration when under inhalant anesthesia. In awake animals, the level should be 0 mm Hg on inhalation and rise to 35- 45 mm Hg on exhalation.

Higher levels of CO, indicate hypoxia and the anesthetist needs to take measures to increase the patient’s oxygen intake and decrease the CO levels. Lower levels indicate that the patient is hyperventilating.

In this article, Samantha Adshead, VTS (Anesthesia & Analgesia) helps explain capnography, including interpreting the readout on a multiparameter monitor.

74
Q

Ionizing radiation is used to create an image in radiography. Which one of the following choices lists another diagnostic modality that uses ionizing radiation?

Computed tomography
Magnetic resonance imaging
Hyperbaric oxygen therapy
Ultrasonography
Bronchoscopy

A

Answer: In computed tomography (CT) ionizing radiation is used to create an image.

During a CT scan, an x-ray beam passes transaxially through the patient. X-ray attenuation (drop off) is measured at sequential sites in a segment of the patient’s
anatomv.

A computer reconstructs the data into a cross-sectional image viewed on a monitor. Computed tomography provides better spatial resolution and radiographic contrast compared to standard radiography. Patients must remain still for a long period of time, necessitating heavy sedation or general anesthesia to achieve proper positioning without motion artifact. Therefore, CT may be too risky in unstable patients.

Click here to see a CT image of a lung lobe torsion in a dog (scroll down).
https://www.vet.upenn.edu/veterinary-hospitals/ryan-veterinary-hospital/services/radiology-diagnostic-imaging/computed-tomography

Link: Computed Tomography in Animals.
https://www.merckvetmanual.com/clinical-pathology-and-procedures/diagnostic-imaging/computed-tomography-in-animals

75
Q

A 4-year old male Manx cat presents to you because the owners found an empty, opened pill vial in the bathroom and the cat vomited. On physical exam, you note ptyalism and facial edema. The cat’s mucous membranes are pale and slightly icteric. You perform a blood smear and detect Heinz bodies in erythrocytes. The cat’s packed cell volume (PCV) is 26% (30-45%). The owners provide you a list of the medications in the medicine cabinet which are acetaminophen (Tylenol), finasteride (Propecia), enalapril (Vasotec), and omeprazole (Prilosec). What treatments should you institute for this cat?

Prednisone and amoxicicilin
Acetylcysteine and S-adenosylmethionine
Activated charcoal and chole blood transfusion
Emesis and methylene blue

A

Answer: Acetylcysteine and S-adenosylmethionine.

Acetaminophen toxicity in cats usually occurs when owners administer the drug, unaware of its significant potential toxicity in cats. In this case, the cat’s clinical signs are most consistent with acetaminophen toxicity based on the Heinz body anemia that is present. Cats can die from oxidative damage and methemoglobinemia within 1-2 days of ingestion. It may also be associated with hepatotoxicity in cats, although this is seen more frequently in dogs.

Recall that cats are particularly sensitive to acetaminophen because they have decreased glucuronyl transferase activity which conjugates acetaminophen to glucuronic acid for excretion. As a result, 50-60 mg (a single tablet) may be fatal for a 4-5 kg cat.

Treatment should consist of toxin removal if possible by inducing emesis in some cases. As the cat in this case is already vomiting, this may not be necessary. Activated charcoal is controversial and should only be given if ingestion occurred within hours and should be administered very carefully in cats due to the risk of aspiration.

The specific antidote is acetylcysteine which binds to some of the reactive metabolites of acetaminophen and increases the availability and synthesis of glutathione. Other treatments may include S-Adenosylmethionine (SAMe) which has hepatoprotective and antioxidant properties. Cimetidine can be given to inhibit the p450 oxidase in the liver and limit formation of toxic metabolites. Ascorbic acid can also be used as an adjunct treatment to bind toxic metabolites. In cats with signs of hypoxemia from severe hemolytic anemia (PCV <20%), a transfusion and further supportive care may be warranted.

76
Q

Which one of the following findings is normal on a complete blood count (CBC) in a cat?

High red cell concentration (polycithemia)
Wanthochromic (yellow) serum
Irregularly shaped red cells (schistocytosis)
Clumped platelets
Lipemic (fatty) serum

A

Answer: Clumped platelets.

Feline platelets (thrombocytes) clump easily. When counting platelets in cat blood, either manually or with an automated cell counter, this must be taken into account.

Feline platelets, essential in hemostasis, are produced in the bone marrow and have a typical lifespan of about 5-7 days. They play a crucial role in primary hemostasis, adhering to damaged endothelial cells and forming a platelet plug. In addition, they contribute to secondary hemostasis by providing a surface for coagulation factor activation. Platelets also have functions beyond clot formation, including roles in immune responses and maintaining vascular integrity. Their production, structure, and function are complex, involving various cytokines and cellular processes. For veterinary professionals, understanding the nuances of feline platelet physiology is crucial, particularly in diagnosing and managing hematologic and vascular disorders.

Feline platelets no clumping, but normal rouleaux formation is clearly demonstrated.
https://www.merckvetmanual.com/multimedia/v4730684

Equine platelets (hard to see-they don’t stain well
https://www.merckvetmanual.com/multimedia/v4730819

Bovine platelets on a blood smear.
https://www.merckvetmanual.com/multimedia/v4730756

Refs: Bassert, Beal and Samples, McCurnin’s Clinical Textbook for Velerinary Technicians, gthed, pp. 373. Duncan and Prasse’s Clinical Pathology, 4t! ed. p.105.

Platelets - Comprehensive Study Guide
Definitions and Production
• Platelets: Anucleate cell fragments produced in the bone marrow from megakaryocytes, essential for hemostasis.
• Production: Stimulated by thrombopoietin from the liver and bone marrow stromal cells. Other cytokines like IL-6, IL-11, GM-CSF also play roles.

Physiopathology
• Formation: Megakaryocytes undergo endomitosis and cytoplasmic maturation, extending proplatelets into vascular sinuses.
• Senescence: Platelets circulate for 5-7 days, undergoing desialylation and apoptosis, removed by liver Kupffer cells and spleen macrophages.

Functions
• Hemostasis: Platelets adhere to damaged endothelium, aggregate, and form a hemostatic plug. They also release granules containing ADP, thromboxane A2, and other factors to promote coagulation.
• Vascular Integrity: Platelet-derived factors maintain endothelial junctions and support angiogenesis.
• Immune Roles: Platelets phagocytize pathogens and modulate immune responses.
Medications
• Thrombopoietin Receptor Agonists: Used in treating refractory thrombocytopenia in humans.

Structure
• Granules:
• Alpha Granules: Contain proteins for adhesion, inflammation, and coagulation.
• Dense Granules: Contain ATP, ADP, serotonin, calcium.
• Lysosomes: Contain hydrolytic enzymes.

77
Q

Hypoventilation can contribute to what adverse outcome during anesthesia?

Atelectasis.
Phlebitis.
Tracheal irritation.
Laryngospasm.
Decubital ulcers.

A

Answer: Atelectasis.

Atelectasis can develop in the lungs when an animal is NOT sufficiently ventilated during anesthesia. You can prevent hypoventilation by gently squeezing the manual reservoir bag to inflate the lungs manually once every 2-5 minutes. This is commonly called ‘bagging’ or ‘sighing.

Tracheal irritation can occur during intubation if the endotracheal tube is too large for
the animal or if the cuff is inflated too much.

Laryngospasms occur during induction. Decubital ulcers are secondary to pressure sores over bony prominences in recumbent animals. Phlebitis is inflammation of the veins.

Here is a good review on hypoventilation, courtesy of the American Animal Hospital
Association (AAHA).

Ref: Bassert, Beal and Samples, McCurnin’s Clinical Textbook for Veterinary Techni 9th ed. p. 1037.

Link: Hypoventilation.
https://www.aaha.org/aaha-guidelines/2020-aaha-anesthesia-and-monitoring-guidelines-for-dogs-and-cats/troubleshooting-anesthetic-complications/hypoventilation/

Hypoventilation in Anesthetic Complications (2020 AAHA Guidelines)
Definitions and Terminology
• Hypoventilation: Inadequate ventilation leading to elevated blood CO2 levels (hypercapnia) and possibly decreased blood oxygen levels (hypoxemia).
• Hypercapnia: Increased partial pressure of CO2 in the blood, typically >45 mm Hg.
• Hypoxemia: Decreased partial pressure of O2 in the blood, typically <80 mm Hg.
Causes of Hypoventilation
• Drug Effects: Anesthetic agents (e.g., opioids, propofol, isoflurane) depress respiratory centers in the brain.
• Airway Obstruction: Due to mucus, foreign bodies, or anatomical abnormalities.
• Mechanical Factors: Improper positioning, equipment malfunction (e.g., ventilator failure), or circuit disconnection.
• Disease Conditions: Pre-existing respiratory or neuromuscular diseases affecting ventilation.
Pathophysiology
• Anesthetic drugs can depress the central respiratory centers, reducing the respiratory rate and tidal volume.
• Reduced muscle tone and airway patency can compromise ventilation.
• Increased CO2 levels lead to respiratory acidosis, affecting cardiovascular function and reducing oxygen delivery to tissues.
• Persistent hypoventilation can result in hypoxemia, leading to tissue hypoxia and potential organ damage.

Clinical Changes and Symptoms
• Increased CO2 Levels: Tachycardia, increased blood pressure initially, followed by bradycardia and hypotension.
• Decreased O2 Levels: Cyanosis, decreased SpO2 readings (<90%).
• Altered Blood Gas Values: Elevated PaCO2 (>45 mm Hg), decreased PaO2 (<80 mm Hg).
Assessment
• Monitoring Tools: Capnography (measuring end-tidal CO2), pulse oximetry (measuring SpO2), blood gas analysis.
• Clinical Signs: Respiratory rate and depth, mucous membrane color, and overall patient demeanor.

Management Strategies
1. Adjust Anesthetic Depth:
• Reduce inhalant anesthetic concentration if excessive.
• Use balanced anesthesia techniques (e.g., combining lower doses of multiple agents).
2. Support Ventilation:
• Manual or mechanical ventilation to ensure adequate tidal volume and respiratory rate.
• Ensure proper positioning to maintain open airways.
3. Treat Underlying Causes:
• Clear airway obstructions, correct equipment malfunctions.
• Administer reversal agents for opioids (e.g., naloxone) or benzodiazepines (e.g., flumazenil) if necessary.
4. Monitor and Correct Blood Gas Abnormalities:
• Supplement oxygen to maintain SpO2 > 95%.
• Correct acid-base imbalances with appropriate fluids and medications.
Medications
• Opioid Reversals:
• Naloxone: 0.01-0.04 mg/kg IV, IM, or SC.
• Side Effects: Reversal of analgesia, potential for acute pain.
• Benzodiazepine Reversals:
• Flumazenil: 0.01 mg/kg IV.
• Side Effects: Seizures in patients with benzodiazepine dependence.
• Supplemental Oxygen:
• Delivered via mask, nasal cannula, or endotracheal tube to improve oxygenation.
Summary for Veterinary Professionals
• Hypoventilation is a critical anesthetic complication requiring prompt recognition and management.
• Monitoring tools like capnography and pulse oximetry are essential for early detection.
• Management includes adjusting anesthetic depth, supporting ventilation, treating underlying causes, and correcting blood gas abnormalities.
• Reversal agents and supplemental oxygen are key treatments.

78
Q

What is the function of the gizzard in the chicken?

Allows predigestion of seeds and grasses similar to cud chewing in cattle
Avian version of gall bladder
Glandular stomach
Grinding function of similar to teeth
Diverticulum (pouch) in the esophagus

A

Answer: Grinding function of similar to teeth.

The gizzard (also called the ventriculus), is the muscular stomach that contains grit and small rocks. It helps birds grind up hard seeds and other foods, in effect, acting like teeth.

The proventriculus is the glandular stomach in birds.

The crop is a diverticulum of the esophagus, used for temporary food storage that empties into the proventriculus. The purpose of the crop is to allow a bird to quickly swallow what food it can, then fly to safety and digest the meal at leisure.

In hand fed pediatric birds (usually valuable parrots), there a many potential problems possible with crop burn, crop puncture or proventricular impaction.

Click here to see a budgerigar with a full crop.
https://en.wikipedia.org/wiki/File:Budgerigar_with_full_crop.jpg

Refs: Bassert, Beal and Samples, McCurnin’s Clinical Textbook for Veterinary Technicians, 9th ed., p. 775 and Wikipedia online.

Link: Pediatric Diseases of Pet Birds
https://www.merckvetmanual.com/exotic-and-laboratory-animals/pet-birds/pediatric-diseases-of-pet-birds

Pediatric Diseases of Pet Birds
1. Aspiration Pneumonia: Caused by improper feeding, leading to respiratory distress. Treat with oxygen, antimicrobials, and supportive care.
2. Bacterial Disease: Gram-negative infections from unsanitary conditions. Symptoms include crop stasis, poor feeding. Treat with antibiotics, supportive care.
3. Yeast Infection: Candida overgrowth, especially with antimicrobial use. Symptoms include crop stasis, oral plaques. Treat with antifungals.
4. Viral Diseases: Includes polyoma virus, avian bornavirus, and circovirus.
5. Foreign Bodies: Diagnosed via radiographs; may require surgical removal.
6. Crop Stasis: Due to poor husbandry/nutrition. Symptoms include a distended crop. Treat with fluid therapy and adjusted feeding practices.
7. Crop Burns: From overheated formula. Treat with antimicrobials, surgery if severe.
8. Esophageal/Pharyngeal Trauma: From improper feeding techniques. Treat with surgical drainage, antimicrobials.
9. Hepatic Lipidosis: Seen in obese, hand-fed chicks. Treat with nutritional adjustments, fluid therapy.
10. Failure to Thrive: Due to genetic, congenital, or husbandry issues. Provide supportive care, possible antimicrobials.
11. Beak Deformities: Corrective measures include manual correction or prosthetics.
12. Splayleg: Deformities due to nutritional deficiencies or improper substrate. Treat with bandages or surgery.
13. Constricted Toe Syndrome: Circulation impeded by fibrous bands. Treat with debridement, moist dressings.
14. Toe Malposition: Correct with early intervention via bandaging.
15. Cryptophthalmia: Reduced palpebral fissures. Surgical correction if vision is compromised.
16. Choanal Atresia: Blocked choanal opening. Treat with surgical creation of an opening.

79
Q

If you take an x-ray of a cat chest at 200 milliamperes for 1/10 th of a second, how milliampere - seconds is the exposure?

200mAs
10 mAs
20 mAs
2000 mAs

A

Answer: 20 mAs.

Together, milliampere-seconds equals mA multiplied by time (mAX sec=mAs), which controls the intensity of an x-ray.

In this question, 200 X 1/10= 20 mAs.

Milliamperes (mA) are the QUANTITY of electrons produced by the x-ray machine and exposure time (sec) is how LONG you expose the animal to these rays.

More mA, or longer exposure time, means a DARKER x-ray.

Refs: Bassert, Beal and Samples, McCurnin’s Clinical Textbook for Veterinary Technicians, gthed. p. 483 and Mosby’s Comprehensive Review for Veterinary Technicians Tighe, Brown, 4th ed. p. 142.

80
Q

What is the function of molar teeth?

A

Answer: Grinding.

Molars grind food. Incisors are for cutting, nibbling food. Canines grip and tear food. The upper fourth premolar and lower first molar in carnivores are used to shear meat and are called the carnassial teeth.

81
Q

What is the normal depth of the gingival sulcus in dogs?

A

Answer: 1-3mm

Normal depth of the gingival sulcus in dogs is 1-3 mm, and in cats is less than 1 mm. The gingival sulcus is the crevice between the tooth and the gumline where plaque collects. It is important to know that this crevice is not very deep in normal cats when you are cleaning teeth.

Dr. Jan Bellows describes the uses of Dental Probes and Explorers
https://www.dvm360.com/view/dental-probes-and-explorers-musts-examination